You are on page 1of 51

Case Digests in LabRel (Batch 1)

1. BITOY JAVIER (DANILO P. JAVIER), Petitioner,


vs.
FLY ACE CORPORATION/FLORDELYN CASTILLO,
G.R. No. 192558               February 15, 2012
PRINICPLE: 
-Illegal dismissal cases is anchored in the existence of employer-employee relationship

-Burden of proof claim in an employer-employee relationship is through substantial evidence and


four fold test.

FACTS: Javier was hired by Fly Ace having the responsibilities of cleaning and arranging the
canned items for delivery to the customers of Fly Ace, in some times he was also asked to
accompany the delivery vehicle (pahinante). He was not issued any identification cards and pay slips
by the company.
In May 2008, Javier was suddenly not allowed to enter the premises of the company upon instruction
of his superior, Mr. Ong. The latter approached his superior and found out that Mr. Ong had been
courting his daughter, Analyn. Once Analyn knew about it, she talked to Mr. Ong and asked him to
spare her father of the trouble, however, Javier was terminated without notice. He was not given the
opportunity to refute the cause of his dismissal. Fly Ace, on the otherhand contends that Javier was
hired under pakyawan basis/payment by result. Thus, Javier filed for illegal dismissal
The Labor Arbiter dismissed the complaint for lack of merit stating Javier’s failure to prove the
employer-employee relationship. The Arbiter further held that having no ID’s and payslips proves
that Javier is not a regular employee, and was only hired in pakyawan basis.
The NLRC on the other hand reversed the LA’s decision, favouring Javier. It rules that the
Javier fulfilled the substantial evidence in the existence of employer-employee relationship. It ruled
that “pakyawan” is a method of compensation and does not define employer-employee relationship.
NLRC also averred that it does not follow that a worker was a contractor or employee, just because
the work he does is not directly related to the employer’s business. 
Thus, this petition under Rule 45
 
ISSUE: WON JAVIER AND FLY ACE HAS EMPLOYER-EMPLOYEE RELATIONSHIP
RULING: The Supreme Court affirmed the Decision of the Labor Arbiter. No employer-employee
relationship could be found.
In an illegal dismissal case, the employer has the burden to prove that its dismissal of an employee
was for a valid cause. However, before a case for illegal dismissal can prosper, an employer-
employee relationship must first be established. The Supreme Court was convinced that the records
and evidence if Javier’s claim was wanting, as he failed to adduce substantial evidence and only
presented self-serving statements. 
While Javier was made to work in the company premises during weekdays arranging and cleaning
grocery items for delivery to clients, no other proof was submitted to fortify his claim. The Court is of
the considerable view that on Javier lies the burden to pass the well-settled tests to determine the
existence of an employer-employee relationship, viz: (1) the selection and engagement of the
employee; (2) the payment of wages; (3) the power of dismissal; and (4) the power to control the
employee’s conduct. Of these elements, the most important criterion is whether the employer
controls or has reserved the right to control the employee not only as to the result of the work but
also as to the means and methods by which the result is to be accomplished. Javier was not able to
persuade the Court that the mentioned four elements exists.  He could not submit competent proof
1avvphi1

that Fly Ace engaged his services as a regular employee; that Fly Ace paid his wages as an
employee, or that Fly Ace could dictate what his conduct should be while at work. In other words,
Javier’s allegations did not establish that his relationship with Fly Ace had the attributes of an
employer-employee relationship on the basis of the above-mentioned four-fold test. Worse, Javier
was not able to refute Fly Ace’s assertion that it had an agreement with a hauling company to
undertake the delivery of its goods. It was also baffling to realize that Javier did not dispute Fly Ace’s
denial of his services’ exclusivity to the company. In short, all that Javier laid down were bare
allegations without corroborative proof.
 
2. SOUTH EAST INTERNATIONAL RATTAN, INC. and/or ESTANISLAO  AGBAY, Petitioners, 1

vs.
JESUS J. COMING, Respondent. G.R. No. 186621               March 12, 2014
PRINCIPLE:  
CONTROL TEST and SUBTANTIAL EVIDENCE

“In any controversy between a laborer and his master, doubts reasonably arising from the evidence
are resolved in favor of the laborer”

FACTS:SEIRI is a domestic corporation engaged in manufacturing and exporting furniture to


various countries. J. COMING who is alleged to be an employee in this case filed a case of illegal
dismissal against SEIRI for being dismissed without lawful cause. J. COMING.
J. COMING alleged that he was a manufacturing employee who was hired by SEIRI in 1984 having
initially compensated in “pakyaw” basis but sometime, had his pay fixed weekly. He was then
dismissed in 2002 under the precept that the company was not doing well financially, and under the
promise that he would be called backed when his services is needed again. J. COMING waited for
over a year but no recall was made.
On the part of SEIRI, it alleged that J. COMING is not included in the list submitted to SSS, and
having V.COMING (j. coming’s brother) executed an affidavit supporting the company’s claim.
Certain persons in the name of Mayol and Apondar issued certifications that J. COMING worked for
them and not for SEIRI. An affidavit of 5 former employees was also submitted by SEIRI denying
J.COMING’s employment to the company.
In the LABOR ARBITER’S DECISION,  he ruled that J. COMING is a regular employee and
termination was illegal. 
This was reversed in the DECISION OF NLRC. It ruled that REINI was incorporated in 1986 while
J.COMING alleged that he was employed, and moreover, when they started to actually operate in
1987, the company was engaged purely on "buying and exporting rattan furniture" hence no
manufacturing employees were hired. The NLRC also rule that J. COMING failed to present a single
payslip, voucher or a copy of a company payroll showing that he rendered service during the period
indicated therein. 
THE CA DID NOT AGREE with NLRC and FOUND AN EMPLOYER-EMPLOYEE
RELATIONSHIP. 
ISSUE: WON THERE IS AN EMPLOYER-EMPLOYEE RELATIONSHIP BETWEEN SEIRI and
J.COMING
RULING: The Supreme Court affirmed the Decision of the CA. It reiterated that there was an
employee-employer relationship 
The Supreme Court found that the comment to the petition filed by SEIRI in the CA, they
emphasized that in the certifications issued by Mayol and Apondar, it was shown that respondent
was employed and working for them in those years he claimed to be working for SEIRI. However, a
reading of the certification by Mayol would show that while the latter claims to have respondent
under his employ in 1997, 1998 and 1999, respondent’s services were not regular and that he works
only if he wants to. Apondar’s certification likewise stated that respondent worked for him since 1999
through his brother Vicente as "sideline" but only after regular working hours and "off and on" basis.
Even assuming the truth of the foregoing statements, these do not foreclose respondent’s regular or
full-time employment with SEIRI. In effect, petitioners suggest that respondent was employed by
SEIRI’s suppliers, Mayol and Apondar but no competent proof was presented as to the latter’s status
as independent contractors.
SEIRI failed to submit proof in their claim that J. COMING was the employee of their suppliers Mayol
and Apondar, who were indeed independent contractors. Clearly, SEIRI failed to discharge their
burden of proving their own affirmative allegation. 

In this case there are doubts arising from the evidence. Thus, the SC held that “In any controversy
between a laborer and his master, doubts reasonably arising from the evidence are resolved in favor
of the labourer”

BERNARD A. TENAZAS, JAIME M. FRANCISCO and ISIDRO G. ENDRACA, Petitioners,


vs.
R. VILLEGAS TAXI TRANSPORT and ROMUALDO VILLEGAS, Respondents.
G.R. No. 192998               April 2, 2014
 
 
PRINCIPLE: CONTROL TEST & SUBSTANTIAL EVIDENCE TO PROVE EMPLOYMENT
REINSTATEMENT IS A REWARD WHEN THERE IS NO STRAINED RELATIONSHIP
BACKWAGES IS A REWARD WHEN THERE IS A STRAINED RELATIONSHIP
 
 
FACTS: Tenazas, Francisco, and Endraca were taxi drivers of R. Villegas Transport, owned by R. Villegas and A.
Villegas. The three filed for illegal dismissal under their own facts:
 
Party PETITIONER’S VERSION R. VILLEGAS TRANSPORT’S  VERSION
Avers that he was dismissed when the taxi he The company admits that Tenaza was an
drove was sideswiped causing a dent. Upon employee, but he was not terminated. The
his return and report to the owners, he was company alleged that he was advised that his
TENAZAS
told to leave and was fired in the instant taxi was up for repairs, and when he was advised
that he could use the taxi again, Tenazas did not
return
Avers that he was dismissed because of the Denies that Francisco is an employee, and denied
FRANCISCO owners’ suspicion that he was organizing a that he was able to drive any of the taxis of the
labor union company
Avers that his dismissal was due to the The company admits that Endraca was an
instance that he fell short of the required employee but only as a spare driver. The allow
boundary during the day when his taxi him to drive whenever a regular driver is not
ENDRACA required immediate repairs and replacement available. Sometime in 2003, he stopped
while it was his turn to drive the taxi. Upon reporting without any reason
his return, he was advised that he was no
longer allowed to drive the taxi. 
 
The Labor Arbiter found no illegal dismissal in the said cases by the according reason:
Party LA’s DECISION
TENAZAS Offered him immediate reinstatement, which offer he refused.
The company denied the existence of an employer-employee relationship. In this situation, the
burden of proof shifts to the complainant to prove the existence of a regular employment.
FRANCISCO Complainant Francisco failed to present evidence of regular employment available to all regular
employees, such as an employment contract, company ID, SSS, withholding tax certificates, SSS
membership and the like.
The company claimed that he was only an extra driver who stopped reporting to queue for
ENDRACA available taxi units which he could drive. In fact, respondents offered him in their Position Paper
on record, immediate reinstatement as extra taxi driver which offer he refused.
The LA further held that the complaint herein is one of actual dismissal. But there was no formal investigations,
no show cause memos, suspension memos or termination memos were never issued. Otherwise stated, there is
no proof of overt act of dismissal committed by herein respondents.
 
The NLRC REVERSED THE DECISION OF LA as it found sufficient evidence to establish employee-employer
relationship, and held that the LA might have not yet seen the Motion to Admit Additional Evidence, as it was filed
late. Thus, considering the evidence the NLRC reversed the Decision

The COURT OF APPEALS AFFIRMED THE DECISION OF NLRC


Party CA’s DECISION
The theory that the two abandoned their work was not believed by the CA because illegal
TENAZAS & dismissal is a plea for continuance of employment. Thus, as they have been admitted to be
ENDRACA employees. CA held that their dismissal was without just cause. They are entitled to back wages
but not separation pay since reinstatement is still possible
FRANCISCO Had not proven the employer employee relationship based on records. Awards are deleted

ISSUE: WON the three drivers are illegally dismissed and entitled of back wages and separation pay

RULING: The Decision of the Court of Appeals is affirmed


 The Supreme Court further stated that it is a repeated rule that in labor cases "the quantum of proof necessary is
substantial evidence, or such amount of relevant evidence which a reasonable mind might accept as adequate to
justify a conclusion." and  "[T]he burden of proof rests upon the party who asserts the affirmative of an issue." In
3  

determining the presence or absence of an employer-employee relationship, the Court has consistently looked for
the following incidents, to wit: (a) the selection and engagement of the employee; (b) the payment of wages; (c)
the power of dismissal; and (d) the employer’s power to control the employee on the means and methods by
which the work is accomplished. The last element, the so-called control test, is the most important element.

Party SC’ s DECISION


Awarded for backwages but not separation pay as reinstatement is still possible. Although the
litigation had engender a certain degree of hostility between the two and the company, the
understandable strain in the parties’ relation would not necessarily rule out reinstatement
TENAZAS &
which would, otherwise, become the rule rather the exception in illegal dismissal cases.
ENDRACA
Strained relations must be demonstrated as a fact, however, to be adequately supported by
evidence—substantial evidence to show that the relationship between the employer and the
employee is indeed strained as a necessary consequence of the judicial controversy
Had not proven the employer employee relationship based on records. Although no particular
form of evidence is required to prove the existence of an employer-employee relationship. Any
competent and relevant evidence to prove the relationship may be admitted. However, he
FRANCISCO
simply relied on his allegation that he was an employee of the company without any other
evidence supporting his claim. Unfortunately for him, a mere allegation in the position paper is
not tantamount to evidence.

4.  
ENRIQUE Y. SAGUN, Petitioner,
vs.
ANZ GLOBAL SEVICES AND OPERATIONS (MANILA), INC., GAY CRUZADA, and PAULA ALCARAZ, Respondents.
G.R. No. 220399 August 22, 2016
 
PRINCIPLE:  COMMENCEMENT OF EMPLOYER-EMPLOYEE RELATIONSHIP
PERFECTION OF CONTRACTS & SUSPENSIVE CONDITIONS
 
FACTS: Sagun was employed at HSBC upon his application to ANZ Global Services. He was offered the position of
customer service representative in June 2011 which he accepted on the same date. Upon acceptance he was
issued a latter of confirmation of offer which provides terms and conditions of his employment which requires
satisfactory results of pre-employment screening. In addition, Sagun was to be placed in probation for 6 months.
However on the date of the start of his probationary status in July 2021, Sagun resigned. On the same date, he was
handed with a letter of retraction which states that the job offer was withdrawn because the company found
inconsistencies in his declared information and documents.

Sagun asserted that his employment contract had been perfected upon his acceptance of the offer in June 2011,
he was deemed employed, one who may only be dismissed with a cause. Thus, he filed for illegal dismissal.

ANZ Global on the other hand averred that there was no employer-employee relationship. They contended that
their offer was conditional and the effectivity of petitioner's employment contract was subject to a term or
period.They claimed that petitioner made material misrepresentations in his job application and interview that
prompted them to withdraw the offer. They pointed out that the discrepancies in his declarations, namely: (a)  that
he only held the position of a Level 1 and not a Level 2 Technical Support Representative at Siemens; and (b)  that
he was terminated for cause due to his absence without official leave (AWOL) and not because of his resignation,
were not satisfactorily explained despite the opportunity accorded to him.

THE LABOR ARBITER DISMISSED THE COMPLAINT, rendering there was no perfection of the employment since
there was a valid cause for the withdrawal

THE NLRC AFFIRMED THE DECISION OF THE LABOR ARBITER

THE COURT OF APPEALS AFFIRMED THE SAME. IT rule that the contract was perfect however, the employment
contract did not commence since respondents did not allow petitioner to begin work due to the
misrepresentations he made in his application form.

ISSUE: WON there is an employer-employee relationship

RULING: THE Supreme Court AFFIRMED THE DECISION, finding no employer-employee relationship.
It further held that a contract is a meeting of minds between two persons whereby one binds himself, with respect
to the other, to give something or to render some service. There is no contract unless the following essential
requisites concur: (a)  consent of the contracting parties; (b) object certain which is the subject matter of the
contract; and (c) cause of the obligation which is established. Contracts undergo three distinct stages. These are
negotiation, perfection or birth, and consummation. The SC held that an employment contract is perfected at the
moment the parties come to agree upon its terms and conditions, and thereafter, concur in the essential elements
thereof. In this case, the Supreme Court agreed with the finding of the CA that there was already a perfected
contract of employment when petitioner signed ANZ's employment offer and agreed to the terms and conditions
that were embodied therein. Nonetheless, the offer of employment extended to petitioner contained several
conditions before he may be deemed an employee of ANZ. Among those conditions for employment was
the "satisfactory completion of any checks (e.g. background, bankruptcy, sanctions and reference checks) that may
be required by ANZ."  Accordingly, petitioner's employment with ANZ depended on the outcome of his
background check, which partakes of the nature of a suspensive condition, and hence, renders the obligation of
the ANZ in this case, conditional. (ART. 1181 of the NCC)
Here, the subject employment contract required a satisfactory completion of petitioner's background check before
he may be deemed an employee of ANZ. Considering, however, that petitioner failed to explain the discrepancies
in his declared information and documents. ANZ's obligations as a would-be employer were held in suspense and
thus, had yet to acquire any obligatory force. Thus, until and unless petitioner complied with the satisfactory
background check, there exists no obligation on the part of ANZ to recognize and fully accord him the rights under
the employment contract

5. LVN PICTURES, INC., petitioner-appellant,


vs.
PHILIPPINE MUSICIANS Guild (FFW) and COURT OF INDUSTRIAL RELATIONS, respondents-
appellee
G.R. No. L-12582             January 28, 1961

PRINCIPLE: CONTROL TEST


 

FACTS: The Philippine Musicians Guild filed a petition before the CIR that sought to be certified
as the sole and exclusive bargaining agency of all musicians working at LVN Pictures, Inc.,
Sampaguita Pictures, Inc., and Premiere Productions, Inc. . The three corporations are engaged
in the making of motion pictures and in the processing and distribution. That said companies
employ musicians for the purpose of making music recordings for title music, background
music, musical numbers, finale music and other incidental music, without which a motion
picture is incomplete. Ninety-five (95%) percent of all the musicians playing for the musical
recordings of said companies are members of the Guild.
In their answers, LVN and Premiere denied that they have any musicians as employees, and
alleged that the musical numbers in the filing of the companies are furnished by independent
contractors. The lower court, however, rejected this pretense and sustained the theory of the
Guild. A reconsideration of the order complained of having been denied by the Court en banc,
LVN Pictures, inc., and Sampaguita Pictures, Inc filed these petitions for review for certiorari.
 ISSUE: WON THE MUSICIANS ARE EMPLOYEES OF THE FILM COMPANIES. 
RULING: THE SUPREME COURT HELD THAT THE MUSICIANS ARE EMPLOYEES OF THE
COMPAIES. 
To determine whether a person who performs work for another is the latter's employee or an
independent contractor, the National Labor Relations relies on 'the right to control' test. Under
this test an employer-employee relationship exist where the person for whom the services are
performed reserves the right to control not only the end to be achieved, but also the manner
and means to be used in reaching the end. 'Notwithstanding that the employees are called
independent contractors', the Board will hold them to be employees where the extent of the
employer's control over them indicates that the relationship is in reality one of employment. 
The right of control of the film company over the musicians is shown (1) by calling the
musicians through 'call slips' in 'the name of the company; (2) by arranging schedules in its
studio for recording sessions; (3) by furnishing transportation and meals to musicians; and (4)
by supervising and directing in detail, through the motion picture director, the performance of
the musicians before the camera, in order to suit the music they are playing to the picture
which is being flashed on the screen. Thus, in the application of Philippine statutes and
pertinent decisions of the United States Courts on the matter to the facts established in this
case, we cannot but conclude that to effectuate the policies of the Act and by virtue of the
'right of control' test, the members of the Philippine Musicians Guild are employees of the
three film companies and, therefore, entitled to right of collective bargaining under Republic
Act No. 875. In view of the fact that the three (3) film companies did not question the union's
majority, the Philippine Musicians Guild is hereby declared as the sole collective bargaining
representative for all the musicians employed by the film companies."

PAGUIO TRANSPORT CORPORATION, petitioner,


vs.
NATIONAL LABOR RELATIONS COMMISSION and WILFREDO MELCHOR, respondents.
G.R. No. 119500 August 28, 1998
 
PRINCIPLE: EMPLOYER-EMPLOYEE RELATIONSHIP IN BOUNDARY SYSTEM FOR TAXIS AND JEEPNEYS
 
FACTS: W. Melchor was hired by Paguio Transport Corp as a taxi driver under the boundary system. He was
engaged to drive the taxi unit assigned to him on a 24-hour schedule per trip every two (2) days, in exchange of
boundary and other deductions imposed on him. In November 1993, complainant allegedly met a vehicular
accident when he accidentally bumped a car which stopped at the intersection even when the traffic light was
green. After he submitted the traffic accident report to the office of respondents, he was allegedly advised to stop
working and have a rest. After several days, he allegedly reported for work only to be told that his service was no
longer needed. Hence, the complaint for illegal dismissal.
 
Paguio Transport in its part maintained that Melchor was not illegally dismissed, there being in the first place no
employer-employee relationship between them. In amplification, it was argued that the element of control  to
determine the existence of such a relationship was lacking. So too, it argued the element of the payment of
compensation. Considering that in lieu of the latter, payment of boundary is instead made allegedly makes the
relationship between them of a "wase-agreement" . Respondents then argued that even if an employer-employee
relationship were to be presumed as present, still complainant's termination arose out of a valid cause and after he
refused to articulate his stand on the investigation being conducted on him. Respondents then harped on the
supposed three occasions when complainant figured in a vehicular accident involving the taxi unit he was driving.
As a result of the alleged compounded damages which the respondents had to shoulder on account of the
supposed reckless driving Melchor, the former was allegedly left with no alternative but to ask complainant's
explanation why he should still be allowed to drive. Complainant, despite several chances, allegedly failed to do
so. 
 
THE NLRC RULED that there was an illegal dismissal.  NLRC uphled the existence of an employer-employee
relationship, citing the Supreme Court ruling that "the relationship created between the parties operating under a
'boundary system' is one of an employer and employee, and not of a lessor and a lessee.". Further, the NLRC
sustained the ruling of the labor arbiter that the private respondent was illegally dismissed, for he "was not
afforded the twin requirements of due process 
 
ISSUE: WON MELCHOR AND PAGUIO TRANSPORT HAD AN EMPLOYER-EMPLOYEE RELATIONSHIP
IF THERE WAS A RELATIONSHIP-WON THERE WAS A JUST CAUSE IN THE DISMISSAL OF MELCHOR

 
RULING:
  FIRST ISSUE: MELCHO AND PAGUIO TRANPOSR HAS EMPLOYER-EMPLOYEE RELATIONSHIP. The
Supreme Court reiterated its Decision in Martinez v. National Labor Relations Commission, which already ruled
that the relationship of taxi owners and taxi drivers is the same as that between jeepney owners and jeepney
drivers under the "boundary system." In both cases, the employer-employee relationship was deemed to exist
which treats the boundary system as that of employer-employee and not of lessor-lessee. In the lease of chattels,
the lessor loses complete control over the chattel leased . In the case of taxi owners/operators and taxi drivers, the
former exercise supervision and control over the latter. The fact that the drivers do not receive fixed wages but get
only the excess of that so-called boundary they pay to the owner/operator is not sufficient to withdraw the
relationship between them from that of employer and employee. The doctrine is applicable in the present case. 
SECOND ISSUE(remedial topic): THERE WAS NO JUST CAUSE. The Supreme Court held that mere
involvement in an accident, absent any showing of fault or recklessness on the part of an employee, is not a valid
ground for dismissal. Paguio Transport, failed to submit any proof to support these allegations. Well-settled is the
rule that the employer has the burden of proving that the dismissal of an employee is for a just cause. The failure
of the employer to discharge this burden means that the dismissal is not justified and that the employee is entitled
to reinstatement and backwages. In this case, the company failed to prove any just or authorized cause for his
dismissal. While Paguio Transport contends that it "submitted and presented material and competent
documentary evidence consisting of police reports of vehicular accidents of taxicab units owned by petitioner and
driven by Melchor, finding the latter at fault for the vehicular accident, however, a careful examination of both
the original Complaint and the Petitioner's Memorandum of Appeal from the labor arbiter's Decision reveals
that said pieces of documentary evidence were not mentioned or included therein,  but were submitted by
petitioner only when he filed his present petition with this Court. Such factual issues cannot be resolved in a
petition for certiorari like the present case, because the Court's review of NLRC decisions is limited to questions of
jurisdiction and grave abuse of discretion.
 

7. TENG vs. PAHAGAC ET AL.


[G.R. No. 169704, November 17, 2010]

Facts:

Albert Teng Fish Trading is engaged in deep sea fishing. Teng claims that he
customarily enters into joint venture agreements with master fishermen who will be in
charge of the management of each fishing venture, including the hiring of the
members of its complement. Teng alleges that the master fishermen hired the
respondent workers as checkers to determine the volume of the fish caught in every
fishing voyage.

Respondent workers filed a complaint for illegal dismissal against Teng before the
NCMB. Respondents alleged that: 

a. Teng hired them without any written employment contract to serve as his eyes
and ears aboard the fishing boats, to classify the fish caught, and to receive
instructions where to unload the catch. 

and

A. That they received regular monthly salaries plus 13th month pay

They asserted that Teng expressed his doubts on the correct volume of the fish caught
and subsequently terminated them. Teng contends that he does not have ay hand in
hiring the respondent workers and that the master fisherman is the one who hired
them. That his role is to supply the equipments and capital.

(Legal Antecedents, baka ma tanong sa recit haha)


*Voluntary Arbitrator ruled in favor of Teng. No EE-ER relationship. Respondents filed
an MR but VA denied the same with teason that the remedy of a MR is not provided
for that party adversely affecter under the Procedural Guidelines in the Conduct of
Voluntary Arbitration Proceedings.

*CA: reversed. There exists an ER-EE relationship between them.

Teng moved to *reconsider the CA’s decision alleging that:

a.The VA’s decision is not subject to MR, hence, the VA’s decision has becom final and
executory

b.No ER-EE relationship.

ISSUE:

W.N there is ER-EE relationship?

RULING:

YES. There exists an ER-EE relationship. While Teng alleged that it was the
maestros who hired the respondent workers, it was his company that issued to the
respondent workers identification cards (IDs) bearing their names as employees and
Teng’s signature as the employer. Generally, in a business establishment, IDs are
issued to identify the holder as a bona fide employee of the issuing entity.

The respondents received uniform amounts of their shares so the arguments of


Teng that it is commission is ridiculous since it would mean that they caught the
same number of fish for 13 years. Moreover, there was element of control as teng
directed the workers on what to do. 

Teng cannot hide behind the fact that Master fisherman hired the workers since that
would mean that Teng committed impermissible labor only contracting. Labor only
contracting is declared prohibited.

There is "labor-only" contracting where the person supplying workers to an employer


does not have substantial capital or investment in the form of tools, equipment,
machineries, work premises, among others, and the workers recruited and placed by
such persons are performing activities which are directly related to the principal
business of such employer. In such cases, the person or intermediary shall be
considered merely as an agent of the employer who shall be responsible to the workers
in the same manner and extent as if the latter were directly employed by him.

8.  
Dy Keh Beng vs. International Labor and Marine Union of thePhilippines, et
al.G.R. No. L-32245, May 25, 1979

FACTS:

A charge of unfair labor practice was filed against Dy Keh Beng, proprietor of a basket
factory, for discriminatory acts within the meaning of Section 4(a), sub-paragraph (1)
and (4). Republic Act No. 875, by dismissing Carlos N. Solano and Ricardo Tudla for
their union activities. After preliminary investigation was conducted, a case was filed
in the Court of Industrial Relations for in behalf of the International Labor and Marine
Union of the Philippines and two of its members, Solano and Tudla.

An employee-employer relationship was found to have existed between Dy Keh Beng


and complainants Tudla and Solano, although Solano was admitted to have worked on
piece basis.
According to Dy Keh Beng, however, Solano was not his employee for the following
reasons:

(1) Solano never stayed long enought at Dy’s establishment;

(2) Solano had to leave as soon as he was through with the

(3) order given him by Dy;

(4) When there were no orders needing his services there was nothing for him to do;

(5) When orders came to the shop that his regular workers could not fill it was then
that Dy went to his address in Caloocan and fetched him for these orders; and

(6) Solano’s work with Dy’s establishment was not continuous.

According to petitioner, these facts show that respondents Solano and Tudla are only
piece workers, not employees under Republic Act 875, where an employee is referred
to as

“shall include any employee and shag not be limited to the employee of a particular
employer unless the Act explicitly states otherwise and shall include any individual
whose work has ceased as a consequence of, or in connection with any current labor
dispute or because of any unfair labor practice and who has not obtained any other
substantially equivalent and regular employment.”

While an employer

“includes any person acting in the interest of an employer, directly or indirectly but
shall not include any labor organization (otherwise than when acting as an employer)
or anyone acting in the capacity of officer or agent of such labor organization.”

ISSUE:

W/N there existed an employee employer relation between petitioner Dy Keh Beng and
the respondents Solano and Tudla .

RULING:
YES. Petitioner really anchors his contention of the non-existence of employee-
employer relationship on the control test. While this Court upholds the control test
under which an employer-employee relationship exists “where the person for whom
the services are performed reserves a right to control not only the end to be achieved
but also the means to be used in reaching such end, ” it finds no merit with
petitioner’s arguments as stated above. It should be borne in mind that the control
test calls merely for the existence of the right to control the manner of doing the work,
not the actual exercise of the right.

Considering the finding by the Hearing Examiner that the establishment of Dy Keh
Beng is “engaged in the manufacture of baskets known as kaing, it is natural to
expect that those working under Dy would have to observe, among others, Dy’s
requirements of size and quality of the kaing. Some control would necessarily be
exercised by Dy as the making of the kaing would be subject to Dy’s specifications.
Parenthetically, since the work on the baskets is done at Dy’s establishments, it can
be inferred that the proprietor Dy could easily exercise control on the men he
employed.

As to the contention that Solano was not an employee because he worked on piece
basis, this Court agrees with the Hearing Examiner that circumstances must be
construed to determine indeed if payment by the piece is just a method of
compensation and does not define the essence of the relation. Units of time … and
units of work are in establishments like respondent just yardsticks whereby to
determine rate of compensation, to be applied whenever agreed upon. We cannot
construe payment by the piece where work is done in such an establishment so as to
put the worker completely at liberty to turn him out and take in another at pleasure.

 
9.

INSULAR LIFE ASSURANCE, CO., LTD. vs. NLRC


G.R. No. 119930 March 12, 1998

FACTS:

Private respondent Pantaleon de los Reyes filed a complaint against petitioner


Insular Life Assurance Co., Ltd., (INSULAR LIFE) for illegal dismissal and nonpayment
of salaries and back wages. This was dismissed by the Labor Arbiter for lack of
jurisdiction after finding that there was no employer-employee relationship between
the two parties. The order was later on reversed by the NLRC on appeal by the private
respondent. Petitioner now seeks for the annulment of the decision of NLRC stating
that the latter erred in acting without jurisdiction and/or with grave abuse of
discretion when it reversed the decision of the Labor Arbiter.

Petitioner contents that they never had any employer-employee relationship for
this was an express agreement between them in the agency contracts and there was a
stipulation therein the de los Reyes was allowed discretion to devise ways and means
to fulfill his obligations as agent and would be paid commission fees based on his
actual output. It further insists that the nature of this work status had already been
resolved in the earlier case of INSULAR LIFE vs. NLRC and Basiao where the Court
declared Basiao to be an independent contractor and not an employee of the
petitioner; therefore, there should be no reason that the same ruling must be applied.

ISSUE:

Is the NLRC correct in ruling that there was an employer-employee relationship


between INSULAR LIFE and de los Reyes?

RULING:

YES. The existence of an employer-employee relationship cannot be negated


by expressly removing it in the management contract and providing therein that
the employee is an independent contractor when the terms of agreement clearly show
otherwise. The employment status of a person is defined and prescribed by law and
not by what the parties say it should be. In determining the status of the management
contract, the four-fold test on employment has to be applied. (1. the selection
and engagement of the employee, 2. payment of wages, 3. the power of
dismissal, and 4. the power of control over the employee's conduct)

One of the terms under the management contract, it was stated that de los
Reyes is obliged to work exclusively for petitioner in life insurance solicitation and was
imposed premium production quotas. He was proscribed from accepting a managerial
or supervisory position in any other office including the government without the
written consent of the petitioner and he could only be promoted to a higher level if he
met certain requirements including the recommendation for promotion by the
petitioners. As found by the NLRC, he exercised administrative functions which were
necessary and beneficial to the business of INSULAR LIFE. This obtaining, there is no
escaping the conclusion that private respondent Pantaleon de los Reyes was an
employee of herein petitioner.

10. TONGKO v. MANUFACTURER LIFE INSURANCE CO. (PHILS.), INC. G.R. No.
167622 January 25, 2011

Facts: 

Manufacturers Life Insurance Co. (Phils.), Inc. (Manulife) is a domestic


corporation engaged in life insurance business. Petitioner started as a manager in a
certain region by virtue of a Career Agent's Agreement which he executed with
Manulife. In the course of his work with respondent, the latter, through a letter,
directed Tongko to exercise particular administrative activities to improve their
operations in the region where the latter was assigned. This is due to the data
collected by Manulife that the region that petitioner is handling is the lowest performer
in terms of recruiting in 2000 and, as of today, continues to remain one of the
laggards in this area. A few months passed, Manulife was still dissatisfied with the
petitioner’s performance despite the letters sent to the latter. This urged Manulife to
terminate Tongko effective 15 days from the receipt of such notice. Tongko then filed a
case with the labor arbiter for illegal dismissal which was denied by the motion of
Manulife arguing that no ER-EE relationship exists based on the agreement contract
they entered from the time Tongko was engaged. Raised to the NLRC, and decided in
favor of Tongko. Manulife appealed to CA, which reversed the decision of NLRC. Hence
the case at bar.

Issue: 
WON there existed an ER-EE relationship between Tongko and Manulife, which
would lead to illegal dismissal by the latter.

Ruling: 

YES. The NLRC, for its part, applied the four-fold test and found the existence
of all the elements and declared Tongko an employee of Manulife. The CA, on the
other hand, found that the element of control as an indicator of the existence of
an employer-employee relationship was lacking in this case. The NLRC and the CA
based their rulings on the same findings of facts but differed on the ruling. NLRC
arrived at its conclusion, first, on the basis of the letter dated November 6, 2001
addressed by De Dios to Tongko. According to the NLRC, the letter contained "an
abundance of directives or orders that are intended to directly affect complainant's
authority and manner of carrying out his functions as Regional Sales Manager." The
NLRC further ruled that the different codes of conduct that were applicable to Tongko
served as the foundations of the power of control wielded by Manulife over Tongko that
is further manifested in the different administrative and other tasks that he was
required to perform. It also found that Tongko was required to render exclusive service
to Manulife, further bolstering the existence of an ER-EE relationship. Finally, the
NLRC ruled that Tongko was integrated into a management structure over which
Manulife exercised control, including the actions of its officers. The NLRC held that
such integration added to the fact that Tongko did not have his own agency belied
Manulife's claim that Tongko was an independent contractor.

SC said that in the instant case, Manulife had the power of control over Tongko
that would make him its employee. Several factors contribute to this conclusion. In an
agreement executed by Tongko and Manulife: 

The Agent hereby agrees to comply with all regulations and requirements of the
Company as herein provided as well as maintain a standard of knowledge and
competency in the sale of the Company's products which satisfies those set by the
Company and sufficiently meets the volume of new business required of Production
Club membership.

Among the company regulations of Manulife are the different codes of conduct
such as the Agent Code of Conduct, Manulife Financial Code of Conduct, and Manulife
Financial Code of Conduct Agreement, which demonstrate the power of control
exercised by the company over Tongko. Thus, with the company regulations and
requirements alone, the fact that Tongko was an employee of Manulife may already be
established. Certainly, these requirements controlled the means and methods by
which Tongko was to achieve the company's goals. More importantly, Manulife's
evidence establishes the fact that Tongko was tasked to perform administrative duties
that establishes his employment with Manulife. Additionally, it must be pointed out
that the fact that Tongko was tasked with recruiting a certain number of agents, in
addition to his other administrative functions, leads to no other conclusion that he
was an employee of Manulife.

And so, due to the findings of the SC regarding the existence of ER-EE
relationship between Tongko and Manulife, the latter is liable to the former for illegal
dismissal.

11. AFP Mutual Benefit Association v. NLRC (G.R. No.  102199)

Facts:

Private respondent Eutiquio Bustamante was an insurance underwriter of petitioner AFP Mutual Benefit
Association until he was dismissed for misrepresentation and for simultaneously selling insurance for
another life insurance company in violation of their agreement. Respondent signed a quitclaim after
receiving his commissions but later on discovered that he was entitled to more than the received
amount. Thus, he filed a complaint with the Office of the Insurance Commissioner but was advised to file
before the Department of Labor, which then ruled in his favor citing that employer-employee
relationship exists between him and petitioner. NLRC tribunal affirmed.

Issue:

Whether or not employer-employee relationship existed between the parties.

Ruling: NO.

The significant factor in determining the relationship of the parties is the presence or absence of
supervisory authority to control the method and the details of performance of the service being
rendered, and the degree to which the principal may intervene to exercise such control. The presence of
such power of control is indicative of an employment relationship, while absence thereof is indicative of
independent contractorship. In other words, the test to determine the existence of independent
contractorship is whether one claiming to be an independent contractor has contracted to do the work
according to his own methods and without being subject to the control of the employer except only as
to the result of the work. Such is exactly the nature of the relationship between petitioner and private
respondent.

Private respondent was free to sell insurance at any time as he was not subject to definite hours or
conditions of work and in turn was compensated according to the result of his efforts. By the nature of
the business of soliciting insurance, agents are normally left free to devise ways and means of
persuading people to take out insurance. There is no prohibition, as contended by petitioner, for private
respondent to work for as long as he does not violate the Insurance Code. Although petitioner could
have, theoretically, disapproved any of private respondent’s transactions, what could be disapproved
was only the result of the work, and not the means by which it was accomplished.
The “control” which the above factors indicate did not sum up to the power to control private
respondent’s conduct in and mode of soliciting insurance. On the contrary, they clearly indicate that the
juridical element of control had been absent in this situation. Thus, the Court is constrained to rule that
no employment relationship had ever existed between the parties.

12. Encyclopaedia Britannica, Inc vs. NLRC, G.R. No. 87098, November 4, 1996; 264 SCRA 1

Posted by Pius Morados on November 10, 2011

(Labor Standards – Inexistence of employer-employee relationship)

Facts: Private respondent was a sales division manager of private petitioner and was in charge of selling
the latter’s products through sales representatives. As compensation, private respondent receive
commissions from the products sold by his agents. After resigning from office to pursue his private
business, he filed a complaint against the petitioner, claiming for non-payment of separation pay and
other benefits.

Petitioner alleged that complainant was not its employee but an independent dealer authorized to
promote and sell its products and in return, received commissions therefrom. Petitioner did not have
any salary and his income from petitioner was dependent on the volume of sales accomplished. He had
his own office, financed the business expense, and maintained his own workforce. Thus petitioner
argued that it had no control and supervision over the complainant as to the manner and means he
conducted his business operations.

The Labor Arbiter ruled that complainant was an employee of the petitioner company. Petioner had
control over the complainant since the latter was required to make periodic reports of his sales activities
to the company.

Issue: Whether or not there exists an employer-employee relationship.

Held: No. Control of employee’s conduct is commonly regarded as the most crucial and determinative
indicator of the presence or absence of an employer-employee relationship. Under this, an employer-
employee relationship exists where the person for whom the services are performed reserves the right
to control not only the end to be achieved, but also the manner and means to be used in reaching that
end.

The fact that petitioner issued memoranda to private respondent and to other division sales managers
did not prove that petitioner had actual control over them. The different memoranda were merely
guidelines on company policies which the sales managers follow and impose on their respective agents.
13. HST Marketing vs. Villastique, G.R. No. 219569, August 17, 2016

FACTS: VIRGILIO O. Villastique was hired by HSY Marketing Ltd. Co. as field driver tasked to deliver
ready-to-wear items and/or general merchandise.

He filed a complaint for illegal dismissal with money claims against HSY Marketing, for allegedly
withholding his salary for his refusal to resign.

Petitioner invoked the defense that since respondent was the one who refused to report for work, he
should be considered as having voluntarily severed his own employment. Thus, his money claims cannot
prosper, as he was not dismissed from the service.

The Labor Arbiter (LA) dismissed the charge of illegal dismissal, finding no evidence to substantiate
respondent’s claim that he was dismissed from his job. There was likewise no evidence submitted by
petitioner that respondent had indeed voluntarily resigned. He ruled that the employer-employee
relationship between the parties should be maintained. Finding however, strained relations between the
parties, he did not order the reinstatement of respondent, and instead directed petitioner to pay him
the amount of P86,580.00 as separation pay.

The National Labor Relations Commission (NLRC) affirmed the finding of the LA. The Court of Appeals
(CA) in turn affirmed in toto the resolution of the NLRC. Petitioner imputes error, in among others, the
award of separation pay. Does the contention find merit?

Ruling: Yes.

The Court likewise upholds the unanimous conclusion of the lower tribunals that respondent had not
been dismissed at all. Other than the latter’s unsubstantiated allegation of having been verbally
terminated from his work, no substantial evidence was presented to show that he was indeed dismissed
or was prevented from returning to his work. In the absence of any showing of an overt or positive act
proving that petitioner had dismissed respondent, the latter’s claim of illegal dismissal cannot be
sustained, as such supposition would be self-serving, conjectural, and of no probative value.

Similarly, petitioner’s claims of respondent’s voluntary resignation and/or abandonment deserve scant
consideration, considering petitioner’s failure to discharge the burden of proving the deliberate and
unjustified refusal of respondent to resume his employment without any intention of returning. It was
incumbent upon petitioner to ascertain respondent’s interest or non-interest in the continuance of his
employment, but to no avail.

Since there is no dismissal or abandonment to speak of, the appropriate course of action is to reinstate
the employee (in this case, herein respondent) without, however, the payment of backwages.

Notably, the reinstatement ordered here should not be construed as a relief proceeding from illegal
dismissal; instead, it should be considered as a declaration or affirmation that the employee may return
to work because he was not dismissed in the first place. For this reason, the Court agrees with petitioner
that the LA, the NLRC, and the CA erred in awarding separation pay in spite of the finding that
respondent had not been dismissed.

Properly speaking, liability for the payment of separation pay is but a legal consequence of illegal
dismissal where reinstatement is no longer viable or feasible. As a relief granted in lieu of reinstatement,
it goes without saying that an award of separation pay is inconsistent with a finding that there was no
illegal dismissal. This is because an employee who had not been dismissed, much less illegally dismissed,
cannot be reinstated. Moreover, as there is no reinstatement to speak of, respondent cannot invoke the
doctrine of strained relations to support his prayer for the award of separation pay. x x x.

In fine, petitioner is ordered to reinstate respondent to his former position without the payment of
backwages. If respondent voluntarily chooses not to return to work, he must then be considered as
having resigned from employment. This is without prejudice, however, to the willingness of both parties
to continue with their former contract of employment or enter into a new one whenever they so desire.
(Perlas-Bernabe, J.; SC 1st Division, HSY Marketing Ltd., Co. vs. Virgilio O. Villastique, G.R. No. 219569,
Aug 17, 2016).
14. COCA COLA VS. CLIMACO DIGEST

DECEMBER 21, 2016 ~ VBDIAZ

G.R. No. 146881             February 5, 2007

COCA COLA BOTTLERS (PHILS.), INC./ERIC MONTINOLA, Manager, Petitioners, 


vs.
DR. DEAN N. CLIMACO, Respondent.

FACTS: Respondent Dr. Dean N. Climaco is a medical doctor  The Retainer Agreement, which began on
January 1, 1988, was renewed annually (original contract was only good for one year). The last one
expired on December 31, 1993. Despite the non-renewal of the Retainer Agreement, respondent
continued to perform his functions as company doctor to Coca-Cola until he received a letter4 dated
March 9, 1995 from petitioner company concluding their retainership agreement effective 30 days from
receipt thereof.

It is noted that as early as September 1992, petitioner was already making inquiries regarding his status
with petitioner company. Petitioner company, however, did not take any action. Hence, respondent
made another inquiry with the DOLE and SSS. Thereafter, respondent inquired from the management of
petitioner company whether it was agreeable to recognizing him as a regular employee. The
management refused to do so.

FILED TWO COMPLAINTS IN THE NLRC: (1)  seeking recognition as a regular employee of petitioner
company and prayed for the payment of all benefits of a regular employee, including 13th Month Pay,
Cost of Living Allowance, Holiday Pay, Service Incentive Leave Pay, and Christmas Bonus; (2) a complaint
for illegal dismissal against petitioner company with the NLRC, Bacolod City.

LABOR ARBITER’S DECISION: Case (1) Dismissed, found that petitioner company lacked the power of
control over respondent’s performance of his duties, and recognized as valid the Retainer Agreement
between the parties; (2) dismissed the case for illegal dismissal in view of the previous finding of Labor
Arbiter that complainant therein, respondent is not an employee of Coca-Cola Bottlers Phils., Inc.
Respondent appealed both decisions to the NLRC, Fourth Division, Cebu City; Dismissed for lack of merit.
MR denied.

APPEAL WITH THE CA: that an employer-employee relationship existed between petitioner company
and respondent after applying the four-fold test.

MR BY PETITONER: The Court of Appeals clarified that respondent was a “regular part-time employee
and should be accorded all the proportionate benefits due to this category of employees of [petitioner]
Corporation under the CBA.” It sustained its decision on all other matters sought to be reconsidered.
Hence, this petition.

ISSUE: whether or not there exists an employer-employee relationship between the parties; The
resolution of the main issue will determine whether the termination of respondent’s employment is
illegal.

HELD: NO employer-employee relationship.

Four-fold test: (1) the selection and engagement of the employee; (2) the payment of wages; (3) the
power of dismissal; and (4) the power to control the employee’s conduct, or the so-called “control test,”
considered to be the most important element.

The Court agrees with the finding of the Labor Arbiter and the NLRC that the circumstances of this case
show that no employer-employee relationship exists between the parties. The Labor Arbiter and the
NLRC correctly found that petitioner company lacked the power of control over the performance by
respondent of his duties. The Labor Arbiter reasoned that the Comprehensive Medical Plan, which
contains the respondent’s objectives, duties and obligations, does not tell respondent “how to
conduct his physical examination, how to immunize, or how to diagnose and treat his patients,
employees of [petitioner] company, in each case.”

petitioner company, through the Comprehensive Medical Plan, provided guidelines merely to ensure
that the end result was achieved, but did not control the means and methods by which respondent
performed his assigned tasks.

Because the company lacks the power of control that the contract provides that respondent shall be
directly responsible to the employee concerned and their dependents for any injury, harm or damage
caused through professional negligence, incompetence or other valid causes of action.

Respondent is not at all further required to just sit around in the premises and wait for an emergency to
occur so as to enable him from using such hours for his own benefit and advantage. In fact, complainant
maintains his own private clinic attending to his private practice in the city, where he services his
patients, bills them accordingly — and if it is an employee of respondent company who is attended to by
him for special treatment that needs hospitalization or operation, this is subject to a special billing.

An employee is required to stay in the employer’s workplace or proximately close thereto that he
cannot utilize his time effectively and gainfully for his own purpose. Such is not the prevailing
situation here.1awphi1. Court finds that the schedule of work and the requirement to be on call for
emergency cases do not amount to such control, but are necessary incidents to the Retainership
Agreement.
The Court also notes that the Retainership Agreement granted to both parties the power to terminate
their relationship upon giving a 30-day notice. Hence, petitioner company did not wield the sole power
of dismissal or termination.

Considering that there is no employer-employee relationship between the parties, the termination of
the Retainership Agreement, which is in accordance with the provisions of the Agreement, does not
constitute illegal dismissal of respondent.

PETITION GRANTED.

15. G.R. No. 129315

October 2, 2000

OSIAS I. CORPORAL, SR., PEDRO TOLENTINO, MANUEL CAPARAS, ELPIDIO LACAP, SIMPLICIO PEDELOS,
PATRICIA NAS, and TERESITA FLORES, petitioners, vs. NATIONAL LABOR RELATIONS COMMISSION, LAO
ENTENG COMPANY, INC. and/or TRINIDAD LAO ONG, respondents.

FACTS: Petitioners, Corporal, Tolentino, Caparas, Lacap, and Pedelos worked as barbers, while Flores and
Nas worked as manicurists in New Look Barber Shop owned by private respondent Lao Enteng Co. Inc..
Petitioner Nas alleged that she also worked as watcher and marketer of private respondent. Petitioners
claim that at the start of their employment, it was a single proprietorship owned and managed by Mr.
Vicente Lao. The children of Vicente Lao organized a corporation which was registered with the SEC as
Lao Enteng Co. Inc. with Trinidad Ong as President. Upon its incorporation, the respondent company
took over the assets, equipment, and properties of the New Look Barber Shop and continued the
business. All the petitioners were allowed to continue working with the new company until when
respondent Ong informed them that the building wherein the New Look Barber Shop was located had
been sold and that their services were no longer needed. Petitioners filed with the NLRC, a complaint for
illegal dismissal, illegal deduction, separation pay, non-payment of 13th month pay, and salary
differentials. Only petitioner Nas asked for payment of salary differentials as she alleged that she was
paid a daily wage of P25.00 throughout her period of employment. The petitioners also sought the
refund of the P1.00 that the respondent company collected from each of them daily as salary of the
sweeper of the barber shop. Private respondent averred that the petitioners were joint venture partners
and were receiving fifty percent commission of the amount charged to customers. Thus, there was no
employer-employee relationship between them and petitioners. And assuming arguendo, that there was
an employer-employee relationship, still petitioners are not entitled to separation pay because the
cessation of operations of the barber shop was due to serious business losses. Moreover, Ong stated
that petitioners they received fifty percent to sixty percent of the gross paid by customers. Trinidad
explained that some of the petitioners were allowed to register with the SSS as employees of Lao
Enteng Company, Inc. only as an act of accommodation. All the SSS contributions were made by
petitioners. Labor Arbiter Cañizares ordered the dismissal of the complaint on the basis of his findings
that the complainants and the respondents were engaged in a joint venture and that there existed no
employer-employee relation between them. The Labor Arbiter also found that the barber shop was
closed due to serious business losses or financial reverses and that the law does not compel the
establishment to pay separation pay to whoever were its employees. On appeal, NLRC affirmed the said
findings of the Labor Arbiter and dismissed the complaint for want of merit. Petitioners fault the NLRC
for arbitrarily disregarding substantial evidence on record showing that petitioners were registered with
the SSS as regular employees of the respondent company.

ISSUE: Whether or not petitioners are regular employees of respondent company.

RULING: Yes. The Labor Arbiter's findings that the parties were engaged in a joint venture is
unsupported by any documentary evidence. It should be noted that aside from the self-serving affidavit
of Trinidad Lao Ong, there were no other evidentiary documents, nor written partnership agreements
presented. We have ruled that even the sharing of proceeds for every job of petitioners in the barber
shop does not mean they were not employees of the respondent company. An independent contractor
is one who undertakes "job contracting", i.e., a person who (a) carries on an independent business and
undertakes the contract work on his own account under his own responsibility according to his own
manner and method, free from the control and direction of his employer or principal in all matters
connected with the performance of the work except as to the results thereof, and (b) has substantial
capital or investment in the form of tools, equipment, machineries, work premises, and other materials
which are necessary in the conduct of the business.11 Juxtaposing this provision vis-à-vis the facts of this
case, we are convinced that petitioners are not "independent contractors". They did not carry on an
independent business. Neither did they undertake cutting hair and manicuring nails, on their own as
their responsibility, and in their own manner and method. The services of the petitioners were engaged
by the respondent company to attend to the needs of its customers in its barber shop. More
importantly, the petitioners, individually or collectively, did not have a substantial capital or investment
in the form of tools, equipment,

work premises and other materials which are necessary in the conduct of the business of the
respondent company. What the petitioners owned were only combs, scissors, razors, nail cutters, nail
polishes, the nippers nothing else. By no standard can these be considered substantial capital necessary
to operate a barber shop. The nature of work performed by were clearly directly related to private
respondent's business of operating barber shops. Respondent company did not dispute that it owned
and operated three (3) barber shops. Hence, petitioners were not independent contractors. The
following elements must be present for an employer-employee relationship to exist: (1) the selection
and engagement of the workers; (2) power of dismissal; (3) the payment of wages by whatever means;
and (4) the power to control the worker's conduct, with the latter assuming primacy in the overall
consideration. As to the "control test", the following facts indubitably reveal that respondent company
wielded control over the work performance of petitioners, in that: (1) they worked in the barber shop
owned and operated by the respondents; (2) they were required to report daily and observe definite
hours of work; (3) they were not free to accept other employment elsewhere but devoted their full time
working in the New Look Barber Shop for all the fifteen (15) years they have worked until April 15, 1995;
(4) that some have worked with respondents as early as in the 1960's; (5) that petitioner Patricia Nas
was instructed by the respondents to watch the other six (6) petitioners in their daily task. Certainly,
respondent company was clothed with the power to dismiss any or all of them for just and valid cause.
Petitioners were unarguably performing work necessary and desirable in the business of the respondent
company. While it is no longer true that membership to SSS is predicated on the existence of an
employee-employer relationship since the policy is now to encourage even the self-employed
dressmakers, manicurists and jeepney drivers to become SSS members, we could not agree with private
respondents that petitioners were registered with the Social Security System as their employees only as
an accommodation. As we have earlier mentioned private respondent showed no proof to their claim
that petitioners were the ones who solely paid all SSS contributions. It is unlikely that respondents
would report certain persons as their workers, pay their SSS premium as well as their wages if it were
not true that they were indeed their employees.

16. Maraguinot vs. NLCR, Del Rosario & Viva Films

Chester Cabalza recommends his visitors to please read the original & full text of the case cited. Xie xie!

ALEJANDRO MARAGUINOT, JR. AND PAUILINO ENERO v. NLRC, VIC DEL ROSARIO, VIVA FILMS
GR No. 120969

Facts:

Maraguinot and Enero were separately hired by Vic Del Rosario under Viva Films as part of the filming
crew. Sometime in May 1992, sought the assistance of their supervisor to facilitate their request that
their salary be adjusted in accordance with the minimum wage law.

On June 1992, Mrs. Cesario, their supervisor, told them that Mr. Vic Del Rosario would agree to their
request only if they sign a blank employment contract. Petitioners refused to sign such document. After
which, the Mr. Enero was forced to go on leave on the same month and refused to take him back when
he reported for work. Mr. Maraguinot on the other hand was dropped from the payroll but was
returned days after. He was again asked to sign a blank employment contract but when he refused, he
was terminated.

Consequently, the petitioners sued for illegal dismissal before the Labor Arbiter. The private
respondents claim the following: (a) that VIVA FILMS is the trade name of VIVA PRODUCTIONS, INC. and
that it was primarily engaged in the distribution & exhibition of movies- but not then making of movies;
(b) That they hire contractors called “producers” who act as independent contractors as that of Vic Del
Rosario; and (c) As such, there is no employee-employer relation between petitioners and private
respondents.

The Labor Arbiter held that the complainants are employees of the private respondents. That the
producers are not independent contractor but should be considered as labor-only contractors and as
such act as mere agent of the real employer. Thus, the said employees are illegally dismissed.
The private respondents appealed to the NLRC which reversed the decision of the Labor Arbiter
declaring that the complainants were project employees due to the ff. reasons: (a) Complainants were
hired for specific movie projects and their employment was co-terminus with each movie project; (b)The
work is dependent on the availability of projects. As a result, the total working hours logged extremely
varied; (c) The extremely irregular working days and hours of complainants work explains the lump sum
payment for their service; and (d) The respondents alleged that the complainants are not prohibited
from working with other movie companies whenever they are not working for the independent movie
producers engaged by the respondents.

A motion for reconsideration was filed by the complainants but was denied by NLRC. In effect, they filed
an instant petition claiming that NLRC committed a grave abuse of discretion in: (a) Finding that
petitioners were project employees; (b) Ruling that petitioners were not illegally dismissed; and (c)
Reversing the decision of the Labor Arbiter.

In the instant case, the petitioners allege that the NLRC acted in total disregard of evidence material or
decisive of the controversy.

Issues:

(a) W/N there exist an employee- employer relationship between the petitioners and the private
respondents.

(b) W/N the private respondents are engaged in the business of making movies.

(c) W/N the producer is a job contractor.

Held:

There exist an employee- employer relationship between the petitioners and the private respondents
because of the ff. reasons that nowhere in the appointment slip does it appear that it was the producer
who hired the crew members. Moreover, it was VIVA’s corporate name appearing on heading of the slip.
It can likewise be said that it was VIVA who paid for the petitioners’ salaries.

Respondents also admit that the petitioners were part of a work pool wherein they attained the status
of regular employees because of the ff. requisites: (a) There is a continuous rehiring of project
employees even after cessation of a project; (b) The tasks performed by the alleged “project employees”
are vital, necessary and indispensable to the usual business or trade of the employer; and (c) However,
the length of time which the employees are continually re-hired is not controlling but merely serves as a
badge of regular employment.

Since the producer and the crew members are employees of VIVA and that these employees’ works deal
with the making of movies. It can be said that VIVA is engaged of making movies and not on the mere
distribution of such.
The producer is not a job contractor because of the ff. reasons: (Sec. Rule VII, Book III of the Omnibus
Rules Implementing the Labor Code.)

a. A contractor carries on an independent business and undertakes the contract work on his own
account under his own responsibility according to his own manner and method, free from the control
and direction of his employer or principal in all matters connected with the performance of the work
except as to the results thereof. The said producer has a fix time frame and budget to make the movies.

b. The contractor should have substantial capital and materials necessary to conduct his business. The
said producer, Del Rosario, does not have his own tools, equipment, machinery, work premises and
other materials to make motion pictures. Such materials were provided by VIVA.

It can be said that the producers are labor-only contractors. Under Article 106 of the Labor Code
(reworded) where the contractor does not have the requisites as that of the job contractors.

17. Calamba Medical Center, Inc. vs National Labor Relations Commission

G.R. No. 176484 – 571 SCRA 585 – Labor Law – Labor Relations – Labor Standards – Strike – Managerial
Employees – Power of Control – Illegal Dismissal

Ronaldo Lanzanas and Merceditha Lanzanas are doctors employed by Calamba Medical Center, Inc. They
are given a retainer’s fee by the hospital as well as shares from fees obtained from patients.

One time, Ronaldo was overheard by Dr. Trinidad talking to another doctor about how low the
admission rate to the hospital. That conversation was reported to Dr. Desipeda who was then the
Medical Director of the hospital.

Eventually Ronaldo was suspended. Ronaldo filed a case for Illegal Suspension in March 1998. In the
same month, the rank and file employees organized a strike against the hospital for unfair labor
practices. Desipeda eventually fired Ronaldo for his alleged participation in the strike, which is not
allowed under the Labor Code for he is a managerial employee. Desipeda also fired Merceditha on the
ground that she is the wife of Ronaldo who naturally sympathizes with him.

The Labor Arbiter ruled that there was no Illegal Suspension for there was no employer-employee
relationship because the hospital has no control over Ronaldo as he is a doctor who even gets shares
from the hospitals earnings.

The National Labor Relations Commission as well as the Court of Appeals reversed the LA.

ISSUE: Whether or not there is an employer-employee relationship?

HELD: Yes. Under the control test, an employment relationship exists between a physician and a hospital
if the hospital controls both the means and the details of the process by which the physician is to
accomplish his task. There is control in this case because of the fact that Desipeda schedules the hours
of work for Ronaldo and his wife.

The doctors are also registered by the hospital under the SSS which is premised on an employer-
employee relationship.
There is Illegal Dismissal committed against Rolando for there was no notice and hearing held. It was
never shown that Rolando joined the strike. But even if he did, he has the right to do so for he is not a
part of the managerial or supervisory employees. As a doctor, their decisions are still subject to
revocation or revision by Desipeda.

There is Illegal Dismissal committed against Merceditha for the ground therefor was not mentioned in
Article 282 of the Labor Code.

When is Control (One of the Four Tests of Employer-Employee Relationship) Absent?

Where a person who works for another does so more or less at his own pleasure and is not subject to
definite hours or conditions of work, and is compensated according to the result of his efforts and not
the amount thereof, the element of control is absent.

AUGUST 12, 2016 BY JAICDN

18. Jardin v. NLRC (G.R. No.  119268)

Facts:

Petitioners were drivers of private respondent’s taxicabs under the boundary system whose earnings
were regularly deducted washing fee for the taxi units. Petitioners decided to form a labor union to
protect their rights and interests on the belief that the deductions made were illegal. Upon learning,
respondent refused to let petitioners drive their taxicabs when they reported for work. Aggrieved,
petitioners filed a complaint for illegal dismissal with the Labor Arbiter but the latter dismissed said
complaint. On appeal, the NLRC tribunal declared that petitioners are employees of private respondent.
On reconsideration however, the decision was reversed by the NLRC tribunal and held that no employer-
employee relationship between the parties exists.

Issue:

Whether or not petitioner taxi drivers are employees of respondent company.

Ruling: YES.

In a number of cases decided by this Court, we ruled that the relationship between jeepney
owners/operators on one hand and jeepney drivers on the other under the boundary system is that of
employer-employee and not of lessor-lessee. In the case of jeepney owners/operators and jeepney
drivers, the former exercise supervision and control over the latter. The management of the business is
in the owner’s hands. The owner as holder of the certificate of public convenience must see to it that
the driver follows the route prescribed by the franchising authority and the rules promulgated as
regards its operation. Now, the fact that the drivers do not receive fixed wages but get only that in
excess of the so-called “boundary” they pay to the owner/operator is not sufficient to withdraw the
relationship between them from that of employer and employee. We have applied by analogy the
doctrine to the relationships between bus owner/operator and bus conductor, auto-calesa
owner/operator and driver, and recently between taxi owners/operators and taxi drivers. Hence,
petitioners are undoubtedly employees of private respondent because as taxi drivers they perform
activities which are usually necessary or desirable in the usual business or trade of their employer.

19. Sonza v. ABS-CBN (G.R. No. 138051)

June 10, 2004 | G.R. No. 138051

Jose "Jay" Y. Sonza, petitioner


ABS-CBN Broadcasting Corp., respondent

FACTS:

In May 1994, ABS-CBN signed an Agreement with the Mel and Jay Management and Development
Corporation (MJMDC). MJMDC agreed to provide Jay Sonza’s services exclusively to ABS-CBN as talent
for radio and television. ABS-CBN agreed to pay for Sonza's services a monthly talent fee of ₱310,000 for
the first year and ₱317,000 for the second and third year of the Agreement.

On April 1, 1996, Sonza wrote a letter wrote a letter to ABS-CBN President Eugenio Lopez III, accusing
ABS-CBN of violating the Agreement.

On April 30, 1996, Sonza filed a complaint before the Department of Labor and Employment (DOLE),
alleging that  that ABS-CBN did not pay his salaries, separation pay, service incentive leave pay, 13th
month pay, signing bonus, travel allowance and amounts due under the Employees Stock Option Plan.

ABS-CBN filed a Motion to Dismiss on the ground that no employer-employee relationship existed
between the parties.

LABOR ARBITER: Dismissed Sonza's complaint for lack of jurisdiction, ruling that because Sonza is a
"talent," he cannot be considered an employee.

NLRC: Dismissed Sonza's Motion for Reconsideration.


COURT OF APPEALS: Affirmed NLRC ruling. The CA ruled that the allegations of Sonza against ABS-CBN
did not constitute a labor dispute because there was no employer-employee relationship to begin with.
If anything, Sonza's allegations constitute an action for breach of contractual obligation, which is
intrinsically a civil dispute to be resolved by a civil court, not the Labor Arbiter or the NLRC.

ISSUE:

Whether Jay Sonza was an employee of ABS-CBN? -- NO.

HELD:

The Supreme Court held that Sonza was not an employee of ABS-CBN. As a "talent," he was an
independent contractor. In coming up with this conclusion, the Court looked at the essential elements of
employer-employee relationship and applied the control test.

(a) Selection and engagement of employee

ABS-CBN engaged Sonza's services to co-host its television and radio programs because of his peculiar
skills, talent and celebrity status. These are indicative, but not conclusive, of an independent contractual
relationship

(b) Payment of wages

The Court held that whatever benefits Sonza enjoyed (SSS, Medicare, 13th month pay) arose from
contract and not because of an employer-employee relationship

(c) Power of dismissal

For violation of any provision of the Agreement, either party may terminate their relationship. Sonza
failed to show that ABS-CBN could terminate his services on grounds other than breach of contract, such
as retrenchment to prevent losses as provided under labor laws.

(d) Power of control

The control test is the most important test our courts apply in distinguishing an employee from an
independent contractor. This test is based on the extent of control the hirer exercises over a worker.
The greater the supervision and control the hirer exercises, the more likely the worker is deemed an
employee. The converse holds true as well – the less control the hirer exercises, the more likely the
worker is considered an independent contractor.

In Sonza's case, ABS-CBN did not exercise control over the means and methods of his work. The Court
found that ABS-CBN was not involved in the actual performance that produced the finished product of
Sonza's work.
Second, the fact that he was subjected to ABS-CBN's rules and standards of performance was not
determinative of control as it was under his contract that he " shall abide with the rules and standards of
performance covering talents of ABS-CBN."

Third, the "exclusivity" clause in the Agreement was not a form of control. In the broadcast industry,
exclusivity is not necessarily the same as control.  The hiring of exclusive talents is a widespread and
accepted practice in the entertainment industry. This practice is not designed to control the means and
methods of work of the talent, but simply to protect the investment of the broadcast station.

Finally, the Supreme Court held that the right of labor to security of tenure as guaranteed in the
Constitution arises only if there is an employer-employee relationship under labor laws. Not every
performance of services for a fee creates an employer-employee relationship. 

 To hold that every person who renders services to another for a fee is an employee - to give meaning to
the security of tenure clause - will lead to absurd results."

20. NELSON V. BEGINO, GENER DEL VALLE, MONINA A VILA-LLORIN AND MA. CRISTINA SUMAYAO,
Petitioners, vs. ABS-CBN CORPORATION (FORMERLY, ABS-CBN BROADCASTING CORPORATION) AND
AMALIA VILLAFUERTE, Respondents.

G.R. No. 199166, 20 April 2015.

PEREZ, J.:

Respondent ABS-CBN, through Respondent Villafuerte, engaged the services of Petitioners as


cameramen, editors or reporters for TV Broadcasting. Petitioners signed regularly renewed Talent
Contracts (3 months - 1 year) and Project Assignment Forms which detailed the duration, budget and
daily technical requirements of a particular project. Petitioners were tasked with coverage of news items
for subsequent daily airings in Respondents’ TV Patrol Bicol Program.

The Talent Contract has an exclusivity clause and provides that nothing therein shall be deemed or
construed to establish an employer-employee relationship between the parties.

Petitioners filed against Respondents a complaint for regularization before the NLRC's Arbitration
branch.

In support of their complaint, Petitioners claimed that they worked under the direct control of
Respondent Villafuerte - they were mandated to wear company IDs, they were provided the necessary
equipment, they were informed about the news to be covered the following day, and they were bound
by the company’s policy on attendance and punctuality.

Respondents countered that, pursuant to their Talent Contracts and Project Assignment Forms,
Petitioners were hired as talents to act as reporters, editors and/or cameramen. Respondents further
claimed they never imposed control as to how Petitioners discharged their duties. At most, they were
briefed regarding the general requirements of the project to be executed.
While the case was pending, Petitioners contracts were terminated, prompting the latter to file a second
complaint for illegal dismissal.

The Arbitration Branch ruled that Petitioners were regular employees, and ordered Respondents to
reinstate the Petitioners.

The NLRC affirmed the ruling, but the CA overturned the decision.

ISSUE: W/N Petitioners are regular employees of Respondents.

RULING: Yes.

Of the criteria to determine whether there is an employer-employee relationship, the so-called "control
test" is generally regarded as the most crucial and determinative indicator of the said relationship.

Under this test, an employer-employee relationship is said to exist where the person for whom the
services are performed reserves the right to control not only the end result but also the manner and
means utilized to achieve the same.

Notwithstanding the nomenclature of their Talent Contracts and/or Project Assignment Forms and the
terms and condition embodied therein, petitioners are regular employees of ABS-CBN.

As cameramen, editors and reporters, it appears that Petitioners were subject to the control and
supervision of Respondents which provided them with the equipment essential for the discharge of their
functions. The exclusivity clause and prohibitions in their Talent Contract were likewise indicative of
Respondents' control over them, however obliquely worded.

Also,the presumption is that when the work done is an integral part of the regular business of the
employer and when the worker does not furnish an independent business or professional service, such
work is a regular employment of such employee and not an independent contractor.
21. Wilhelmina Orozco vs Court of Appeals (2005)

457 SCRA 700  – Labor Law – Labor Standards –  Bond Requirement When Employer Appeals in a Labor
Case

Wilhelmina Orozco was hired as a writer by the Philippine Daily Inquirer (PDI) in 1990. She was the
columnist of “Feminist Reflections” under the Lifestyle section of the publication. She writes on a weekly
basis and on a per article basis (P250-300/article).

In 1991, Magsanoc as the editor-in-chief sought to improve the Lifestyle section of the paper. She said
there were too many Lifestyle writers and that it was time to reduce the number of writers. Orozco’s
column was eventually dropped.

Orozco filed for a case for Illegal Dismissal against PDI and Magsanoc. Orozco won in the Labor Arbiter.
The LA ruled that there exists an employer-employee relationship between PDI and Orozco hence
Orozco is entitled to receive backwages, reinstatement, and 13th month pay.

PDI appealed to the National Labor Relations Commission. The NLRC denied the appeal because of the
failure of PDI to post a surety bond as required by Article 223 of the Labor Code. The Court of Appeals
reversed the NLRC.

ISSUE: Whether or not there exists an employer-employee relationship between PDI and Orozco.
Whether or not PDI’s appeal will prosper.

HELD: Under Article 223 of the Labor Code:

ART. 223.  Appeal.  – Decisions, awards or orders of the Labor Arbiter are final and executory unless
appealed to the Commission by any or both parties within ten (10) calendar days from receipt of such
decisions, awards, or orders.
In case of a judgment involving a monetary award, an appeal by the employer may be perfected only
upon the posting of a cash or surety bond issued by a reputable bonding company duly accredited by
the Commission in the amount equivalent to the monetary award in the judgment appealed from.

The requirement that the employer post a cash or surety bond to perfect its/his appeal is apparently
intended to assure the workers that if they prevail in the case, they will receive the money judgment in
their favor upon the dismissal of the employer’s appeal. It was intended to discourage employers from
using an appeal to delay, or even evade, their obligation to satisfy their employees’ just and lawful
claims.

But in this case, this principle is relaxed by the Supreme Court considering the fact that the Labor
Arbiter, in ruling that the Orozco is entitled to backwages, did not provide any computation.

The case is then remanded to the Labor Arbiter for the computation. This necessarily pended the
resolution of the other issue of whether or not there exists an employer-employee relationship between
PDI and Orozco.

22. Television and Production Exponents, Inc. v. Servana January 28, 2008 G.R. No. 167648 542

SCRA 578

Facts: Television and Production Exponents (TAPE) is a domestic corporation engaged in the

production of television programs, such as the long-running variety program, “Eat Bulaga”. Servana

had served as a security guard for TAPE. Respondent filed a complaint for illegal dismissal and

non-payment of benefits against TAPE. He alleged that he was first connected with Agro-

Commercial Security Agency but was later absorbed by TAPE as a regular company guard.

On March 2, 2000, respondent received a memorandum informing him of his impending dismissal

on account of TAPE’s decision to contract the services of a professional security agency. At the time

of his termination, respondent was receiving a monthly salary P6,000. Servana contended that his

dismissal was undertaken without due process and violation of existing labor laws, aggravated by

non-payment of separation pay. He insisted that he was a regular employee having been engaged to

perform an activity that is necessary and desirable to TAPE’s business for 13 years.

TAPE contended that there is no employer-employee relationship between the parties. TAPE

engaged respondent’s services, as part of the support group to provide security service and it was

agreed that complainant would render his services until such time that respondent company shall

have engaged the services of a professional security agency. TAPE started negotiations for the

engagement of a professional security agency , the Sun Shield Security Agency.


TAPE averred that respondent was an independent contractor falling under the talent group category

and was working under a special arrangement which is recognized in the industry.

Issue: WON the Servana is an independent contractor.

Ruling:

TAPE failed to establish that respondent is an independent contractor.!

Jurisprudence is abound woith casesn that recite the factors to be considered in determining the

existence of employer-employee relationship, namely:

a. The selection and engagement of the employee!

Respondent was first connected with Agro-Commercial Security Agency, which assigned him to

assist TAPE in its live productions. When the security agency’s contract with RPN-9 expired,

respondent was absorbed by TAPE , or in the latter’s language, “retained as talent”. Clearly,

respondent was hired by TAPE. Respondent presented his identification card. It has been in" held

that in business establishment, an identification card is usually provided not just as a security

measure but to mainly identify the holder thereof as a bona fide employee of the firm who issues it.

b. The payment of wages

Respondent claims to have been receiving P5,444.44 as his monthly salary while TAPE prefers to

designate such amount as talent fees. "Wages, as defined in the Labor Code, are remuneration or

earnings, however designated, capable of being expressed in terms of money, whether fixed or

ascertained on a time, task, piece or commission basis, or other method of calculating the same,

which is payable by an employer to an employee under a written or unwritten contract of

employment for work done or to be done, or for service rendered or to be rendered.

c. The power of dismissal

The Memorandum informing respondent of the discontinuance of his service proves that TAPE had

the power to dismiss respondent.

d. The employer’s power to control the employee with respect to the means and method by

which the work is to be accomplished.!

Control is manifested in the bundy cards submitted by respondent in evidence. He was required to
report daily and observe definite work hours.

23. ANGELINA FRANCISCO,  Petitioner, versus NATIONAL LABOR RELATIONS COMMISSION, KASEI
CORPORATION, SEIICHIRO TAKAHASHI, TIMOTEO ACEDO, DELFIN LIZA, IRENE BALLESTEROS, TRINIDAD
LIZA and RAMON ESCUETA, Respondents., G.R. No. 170087, 2006 Aug 31.

FACTS:

1995, Petitioner was hired by Kasei Corporation during its incorporation stage.  She was designated as
Accountant and Corporate Secretary and was assigned to handle all the accounting needs of the
company.  She was also designated as Liaison Officer to the City of Makati to secure business permits,
construction permits and other licenses for the initial operation of the company.

Although she was designated as Corporate Secretary, she was not entrusted with the corporate
documents; neither did she attend any board meeting nor required to do so.  She never prepared any
legal document and never represented the company as its Corporate Secretary. 1996, petitioner was
designated Acting Manager. Petitioner was assigned to handle recruitment of all employees and
perform management administration functions; represent the company in all dealings with government
agencies, especially with the BIR, SSS and in the city government of Makati; and to administer all other
matters pertaining to the operation of Kasei Restaurant which is owned and operated by Kasei
Corporation. January 2001, petitioner was replaced by a certain Liza R. Fuentes as Manager. Kasei
Corporation reduced her salary, she was not paid her mid-year bonus allegedly because the company
was not earning well.  On October 2001, petitioner did not receive her salary from the company.  She
made repeated follow-ups with the company cashier but she was advised that the company was not
earning well. Eventually she was informed that she is no longer connected with the company.

Since she was no longer paid her salary, petitioner did not report for work and filed an action for
constructive dismissal before the labor arbiter. Private respondents averred that petitioner is not an
employee of Kasei Corporation.  They alleged that petitioner was hired in 1995 as one of its technical
consultants on accounting matters and act concurrently as Corporate Secretary.  As technical consultant,
petitioner performed her work at her own discretion without control and supervision of Kasei
Corporation. Petitioner had no daily time record and she came to the office any time she wanted and
that her services were only temporary in nature and dependent on the needs of the corporation.

The Labor Arbiter found that petitioner was illegally dismissed, NLRC affirmed with modification the
Decision of the Labor Arbiter. On appeal, CA reversed the NLRC decision. CA  denied petitioner’s MR,
hence, the present recourse.

ISSUES:
1. WON there was an employer-employee relationship between petitioner and private respondent; and
if in the affirmative,
2. Whether petitioner was illegally dismissed.
RULING:

1. Generally, courts have relied on the so-called right of control test where the person for whom
the services are performed reserves a right to control not only the end to be achieved but also
the means to be used in reaching such end. In addition to the standard of right-of-control, the
existing economic conditions prevailing between the parties, like the inclusion of the employee
in the payrolls, can help in determining the existence of an employer-employee relationship.

There are instances when, aside from the employer’s power to control the employee, economic realities
of the employment relations help provide a comprehensive analysis of the true classification of the
individual, whether as employee, independent contractor, corporate officer or some other capacity.

It is better, therefore, to adopt a two-tiered test involving: (1) the employer’s power to control; and (2)
the economic realities of the activity or relationship.

The control test means that there is an employer-employee relationship when the person for whom the
services are performed reserves the right to control not only the end achieved but also the manner and
means used to achieve that end.

There has to be analysis of the totality of economic circumstances of the worker. Thus, the
determination of the relationship between employer and employee depends upon the circumstances of
the whole economic activity, such as: (1) the extent to which the services performed are an integral part
of the employer’s business; (2) the extent of the worker’s investment in equipment and facilities; (3) the
nature and degree of control exercised by the employer; (4) the worker’s opportunity for profit and loss;
(5) the amount of initiative, skill, judgment or foresight required for the success of the claimed
independent enterprise; (6) the permanency and duration of the relationship between the worker and
the employer; and (7) the degree of dependency of the worker upon the employer for his continued
employment in that line of business. The proper standard of economic dependence is whether the
worker is dependent on the alleged employer for his continued employment in that line of business. By
applying the control test, it can be said that petitioner is an employee of Kasei Corporation because she
was under the direct control and supervision of Seiji Kamura, the corporation’s Technical Consultant.
She reported for work regularly and served in various capacities as Accountant, Liaison Officer, Technical
Consultant, Acting Manager and Corporate Secretary, with substantially the same job functions, that is,
rendering accounting and tax services to the company and performing functions necessary and desirable
for the proper operation of the corporation such as securing business permits and other licenses over an
indefinite period of engagement. Respondent corporation had the power to control petitioner with the
means and methods by which the work is to be accomplished.
Under the economic reality test, the petitioner can also be said to be an employee of respondent
corporation because she had served the company for 6 yrs. before her dismissal, receiving check
vouchers indicating her salaries/wages, benefits, 13th month pay, bonuses and allowances, as well as
deductions and Social Security contributions from. When petitioner was designated General Manager,
respondent corporation made a report to the SSS.  Petitioner’s membership in the SSS evinces the
existence of an employer-employee relationship between petitioner and respondent corporation. The
coverage of Social Security Law is predicated on the existence of an employer-employee relationship.

1. The corporation constructively dismissed petitioner when it reduced her.  This amounts to an
illegal termination of employment, where the petitioner is entitled to full backwages

A diminution of pay is prejudicial to the employee and amounts to constructive dismissal.  Constructive
dismissal is an involuntary resignation resulting in cessation of work resorted to when continued
employment becomes impossible, unreasonable or unlikely; when there is a demotion in rank or a
diminution in pay; or when a clear discrimination, insensibility or disdain by an employer becomes
unbearable to an employee. Petition is GRANTED. 

24. WPP MARKETING V. GALERA (G.R. NO. 169207; MARCH 25, 2010)

CASE DIGEST: WPP MARKETING COMMUNICATIONS, INC., JOHN STEEDMAN, MARK WEBSTER, and
NOMINADA LANSANG, Petitioners, v. JOCELYN M. GALERA, Respondent.

FACTS: Petitioner is Jocelyn Galera (GALERA), an American citizen who was recruited from the United
States of America by private respondent John Steedman, Chairman-WPP Worldwide and Chief Executive
Officer of Mindshare, Co., a corporation based in Hong Kong, China, to work in the Philippines for private
respondent WPP Marketing Communications, Inc. (WPP), a corporation registered and operating under
the laws of Philippines.

Employment of GALERA with private respondent WPP became effective on September 1, 1999 solely on
the instruction of the CEO and upon signing of the contract, without any further action from the Board
of Directors of private respondent WPP.

Four months had passed when private respondent WPP filed before the Bureau of Immigration an
application for petitioner GALERA to receive a working visa, wherein she was designated as Vice
President of WPP. Petitioner alleged that she was constrained to sign the application in order that she
could remain in the Philippines and retain her employment.

On December 14, 2000, petitioner GALERA alleged she was verbally notified by private respondent
STEEDMAN that her services had been terminated from private respondent WPP. A termination letter
followed the next day. Thus, a complaint for illegal dismissal was filed against WPP.

The LA held that WPP, Steedman, Webster, and Lansang liable for illegal dismissal and damages. Arbiter
Madriaga stated that Galera was not only illegally dismissed but was also not accorded due process. The
NLRC reversed the LA decision. The NLRC stressed that Galera was WPPs Vice-President, and therefore,
a corporate officer at the time she was removed by the Board of Directors. Such being the case, the
imperatives of law require that we hold that the Arbiter below had no jurisdiction over Galeras case as,
again, she was a corporate officer at the time of her removal.

On appeal, the CA reversed the NLRC decision. It ruled that a person could be considered a "corporate
officer" only if appointed as such by a corporations Board of Directors, or if pursuant to the power given
them by either the Articles of Incorporation or the By-Laws.

ISSUE: Does the LA have jurisdiction over the case?

HELD: Under Section 25 of the Corporation Code, the corporate officers are the president, secretary,
treasurer and such other officers as may be provided in the by-laws.

An examination of WPPs by-laws resulted in a finding that Galeras appointment as a corporate officer
(Vice-President with the operational title of Managing Director of Mindshare) during a special meeting
of WPP's Board of Directors is an appointment to a non-existent corporate office. WPPs by-laws
provided for only one Vice-President. At the time of Galeras appointment on 31 December 1999, WPP
already had one Vice-President in the person of Webster. Galera cannot be said to be a director of WPP
also because all five directorship positions provided in the by-laws are already occupied. Finally, WPP
cannot rely on its Amended By-Laws to support its argument that Galera is a corporate officer. The
Amended By-Laws provided for more than one Vice-President and for two additional directors. Even
though WPPs stockholders voted for the amendment on 31 May 2000, the SEC approved the
amendments only on 16 February 2001. Galera was dismissed on 14 December 2000. WPP, Steedman,
Webster, and Lansang did not present any evidence that Galeras dismissal took effect with the action of
WPP's Board of Directors.

Galera being an employee, then the Labor Arbiter and the NLRC have jurisdiction over the present
case. ***

WPPs dismissal of Galera lacked both substantive and procedural due process. Apart from Steedman's
letter dated 15 December 2000 to Galera, WPP failed to prove any just or authorized cause for Galeras
dismissal.

The law further requires that the employer must furnish the worker sought to be dismissed with two
written notices before termination of employment can be legally effected: (1) notice which apprises the
employee of the particular acts or omissions for which his dismissal is sought; and (2) the subsequent
notice which informs the employee of the employers decision to dismiss him. Failure to comply with the
requirements taints the dismissal with illegality. WPPs acts clearly show that Galeras dismissal did not
comply with the two-notice rule. ***

The employment permit must be acquired prior to employment.


The law and the rules are consistent in stating that the employment permit must be acquired prior to
employment. The Labor Code states: "Any alien seeking admission to the Philippines for employment
purposes and any domestic or foreign employer who desires to engage an alien for employment in the
Philippines shall obtain an employment permit from the Department of Labor."

Galera cannot come to this Court with unclean hands. To grant Galeras prayer is to sanction the
violation of the Philippine labor laws requiring aliens to secure work permits before their employment.
We hold that the status quo must prevail in the present case and we leave the parties where they are.
Hence, Galera is not entitled to monetary awards. This ruling, however, does not bar Galera from
seeking relief from other jurisdictions. GRANTED.

25. MATLING INDUSTRIAL AND COMMERCIAL CORPORATION, RICHARD K. SPENCER, CATHERINE


SPENCER, AND ALEX MANCILLA, petitioners, vs. RICARDO R. COROS, respondent.

G.R. No. 157802. October 13, 2010.

FACTS:

After his dismissal by Matling as its Vice President for Finance and Administration, the respondent filed a
complaint for illegal suspension and illegal dismissal against Matling and some of its corporate officers
(petitioners) in the NLRC, Sub-Regional Arbitration Branch XII, Iligan City.

The petitioners moved to dismiss the complaint, raising the ground, among others, that the complaint
pertained to the jurisdiction of the Securities and Exchange Commission (SEC) due to the controversy
being intra-corporate inasmuch as the respondent was a member of Matling’s Board of Directors aside
from being its Vice President for Finance and Administration prior to his termination.

The LA granted the petitioners’ motion to dismiss, ruling that the respondent was a corporate officer
because he was occupying the position of Vice President for Finance and Administration and at the same
time was a Member of the Board of Directors of Matling; and that, consequently, his removal was a
corporate act of Matling and the controversy resulting from such removal was under the jurisdiction of
the SEC.

The NLRC set aside the dismissal, concluding that the respondent’s complaint for illegal dismissal was
properly cognizable by the LA, not by the SEC, because he was not a corporate officer by virtue of his
position in Matling, albeit high ranking and managerial, not being among the positions listed in Matling’s
Constitution and By-Laws.

The petitioners elevated the issue to the CA by petition for certiorari. The CA dismissed the petition for
certiorari explaining that “for a position to be considered as a corporate office, the position must, if not
listed in the by-laws, have been created by the corporation’s board of directors, and the occupant
thereof appointed or elected by the same board of directors or stockholders.”

ISSUE:
Whether or not respondent was a corporate officer

Whether the LA or the RTC has jurisdiction over his complaint for illegal dismissal

RULING:

As a rule, the illegal dismissal of an officer or other employee of a private employer is properly
cognizable by the LA. Where the complaint for illegal dismissal concerns a corporate officer, however,
the controversy falls under the jurisdiction of the Securities and Exchange Commission (SEC), because
the controversy arises out of intra-corporate or partnership relations between and among stockholders,
members, or associates, or between any or all of them and the corporation, partnership, or association
of which they are stockholders, members, or associates, respectively; and between such corporation,
partnership, or association and the State insofar as the controversy concerns their individual franchise
or right to exist as such entity; or because the controversy involves the election or appointment of a
director, trustee, officer, or manager of such corporation, partnership, or association. Such controversy,
among others, is known as an intra-corporate dispute.

Effective on August 8, 2000, upon the passage of Republic Act No. 8799, otherwise known as The
Securities Regulation Code, the SEC’s jurisdiction over all intra-corporate disputes was transferred to the
RTC.

Under Sec. 25 of the Corporation Code, a position must be expressly mentioned in the By-Laws in order
to be considered as a corporate office. Thus, the creation of an office pursuant to or under a By-Law
enabling provision is not enough to make a position a corporate office.

Here, respondent’s position of Vice President for Finance and Administration was not expressly
mentioned in the By-Laws; neither was the position of Vice President for Finance and Administration
created by Matling’s Board of Directors. Lastly, the President, not the Board of Directors, appointed him.

In order to determine whether a dispute constitutes an intra-corporate controversy or not, the Court
considers two elements instead, namely: (a) the status or relationship of the parties; and (b) the nature
of the question that is the subject of their controversy.

The criteria for distinguishing between corporate officers who may be ousted from office at will, on one
hand, and ordinary corporate employees who may only be terminated for just cause, on the other hand,
do not depend on the nature of the services performed, but on the manner of creation of the office. In
the respondent’s case, he was supposedly at once an employee, a stockholder, and a Director of
Matling. The circumstances surrounding his appointment to office must be fully considered to
determine whether the dismissal constituted an intra-corporate controversy or a labor termination
dispute.

Obviously enough, the respondent was not appointed as Vice President for Finance and Administration
because of his being a stockholder or Director of Matling. He had started working for Matling on
September 8, 1966, and had been employed continuously for 33 years until his termination on April 17,
2000, first as a bookkeeper, and his climb in 1987 to his last position as Vice President for Finance and
Administration had been gradual but steady.

Even though he might have become a stockholder of Matling in 1992, his promotion to the position of
Vice President for Finance and Administration in 1987 was by virtue of the length of quality service he
had rendered as an employee of Matling. His subsequent acquisition of the status of
Director/stockholder had no relation to his promotion. Besides, his status of Director/stockholder was
unaffected by his dismissal from employment as Vice President for Finance and Administration.

26. NICANOR F. MALCABA, CHRISTIAN C. NEPOMUCENO, AND LAURA MAE FATIMA F. PALIT-
ANG, Petitioners, v. PROHEALTH PHARMA PHILIPPINES, INC., GENEROSO R. DEL CASTILLO, JR., AND
DANTE M. BUSTO, Respondents.

G.R. No. 209085, June 6, 2018, Third Division Decision, Justice Leonen

Facts:

At the time of his alleged dismissal, petitioner Malcaba was the President of respondent corporation. As
a consequence, petitioner questioned his dismissal and filed a Complaint for Illegal Dismissal before the
Labor Arbiter.

When the case was elevated before the Court of Appeals, it dismissed Malcaba’s complaint for lack of
jurisdiction since Malcaba, being a corporate officer, should have filed his complaint with the regular
court and not with the labor arbiter.

Issue:

Whether or not the labor arbiter has jurisdiction over petitioner Malcaba’s complaint.

Ruling:

Under Section 25 of the Corporation Code, the President of a corporation is considered a corporate
officer. The dismissal of a corporate officer is considered an intra-corporate dispute, not a labor dispute.
Thus, a corporate officer’s dismissal is always a corporate act, or an intracorporate controversy, and the
nature is not altered by the reason or wisdom with which the Board of Directors may have in taking such
action. Also, an intracorporate controversy is one which arises between a stockholder and the
corporation. There is no distinction, qualification, nor any exemption whatsoever. The provision is broad
and covers all kinds of controversies between stockholders and corporations.
The clear weight of jurisprudence clarifies that to be considered a corporate officer, the office must be
created by the charter of the corporation, and second, the officer must be elected by the board of
directors or by the stockholders.

Petitioner Malcaba was an incorporator of the corporation and a member of the Board of Directors.
Respondent corporation’s By-Laws creates the office of the President. That foundational document also
states that the President is elected by the Board of Directors.

Finding that petitioner Malcaba is the President of respondent corporation and a corporate officer, any
issue on his alleged dismissal is beyond the jurisdiction of the Labor Arbiter or the National Labor
Relations Commission. Their adjudication on his money claims is void for lack of jurisdiction. As a matter
of equity, petitioner Malcaba must, therefore, return all amounts received as judgment award pending
final adjudication of his claims. The Court’s dismissal of petitioner Malcaba’s claims, however, is without
prejudice to his filing of the appropriate case in the proper forum.

27. Republic v. Asiapro Cooperative (G.R. No.  172101)

Facts:

Respondent Asiapro Cooperative is composed of owners-members with primary objectives of providing


them savings and credit facilities and livelihood services. In discharge of said objectives, Asiapro entered
into several service contracts with Stanfilco. Sometime later, the cooperative owners-members
requested Stanfilco’s help in registering them with SSS and remitting their contributions. Petitioner SSS
informed Asiapro that being actually a manpower contractor supplying employees to Stanfilco, it must
be the one to register itself with SSS as an employer and remit the contributions. Respondent
continuously ignoring the demand of SSS the latter filed before the SSC. Asiapro alleges that there exists
no employer-employee relationship between it and its owners-members. SSC ruled in favor of SSS. On
appeal, CA reversed the decision.

Issue:

Whether or not there is employer-employee relationship between Asiapro and its owners-members.

Ruling: YES.

In determining the existence of an employer-employee relationship, the following elements are


considered: (1) the selection and engagement of the workers; (2) the payment of wages by whatever
means; (3) the power of dismissal; and (4) the power to control the worker‘s conduct, with the latter
assuming primacy in the overall consideration. All the aforesaid elements are present in this case.

First. It is expressly provided in the Service Contracts that it is the respondent cooperative which has the
exclusive discretion in the selection and engagement of the owners-members as well as its team leaders
who will be assigned at Stanfilco.
Second. It cannot be doubted then that those stipends or shares in the service surplus are indeed wages,
because these are given to the owners-members as compensation in rendering services to respondent
cooperative‘s client, Stanfilco.

Third. It is also stated in the above-mentioned Service Contracts that it is the respondent cooperative
which has the power to investigate, discipline and remove the owners-members and its team leaders
who were rendering services at Stanfilco.

Fourth. In the case at bar, it is the respondent cooperative which has the sole control over the manner
and means of performing the services under the Service Contracts with Stanfilco as well as the means
and methods of work. Also, the respondent cooperative is solely and entirely responsible for its owners-
members, team leaders and other representatives at Stanfilco. All these clearly prove that, indeed, there
is an employer-employee relationship between the respondent cooperative and its owners-members.

28. Pasay City Alliance Church v. Benito

G.R. 226908, November 28, 2019

DOCTRINE:

               The termination of a religious minister’s engagement at a local church due to administrative


lapses, when it relates to the perceived effectivity of a minister as a charismatic leader of a
congregation, is a prerogative best left to the church affected by such choice. If a religious association
enacts guidelines that reserve the right to transfer or reassign its licensed ministers according to what it
deems to be for a particular congregation, ministry or undertaking in pursuit of its mission, then the
State cannot validly interfere.

FACTS:

               Fe Benito was a licensed Christian minister of the Christian and Missionary Alliance Churches of
the Philippines (CAMACOP), and was appointed as head of Membership and Evangelism Ministry of
Pasay City Alliance Church (PCAC), one of the local churches of CAMACOP. She served as a pastor in
PCAC without a written contract. Under the CAMACOP’s Amended Local Church Administrative and
Ministry Guidelines, the term of ministry shall be determined mutually with the church and confirmed
by the District Ministry Supervisor. In the absence of a contract, the pastor should tender a courtesy
resignation every year. Pastors who were not reappointed to their previous posts were assigned in
different position, local church or specialized ministry.  Benito complied with this rule, and tendered her
resignation every year. She was reappointed as head of the said ministry for consecutive years.
However, in 2013, Benito was informed of the non-extension of her engagement as PCAC’s head for
Membership and Evangelism. This decision was based on the evaluation of her poor performance as the
ministry head.
               Aggrieved, Benito filed a complaint for illegal dismissal before the Labor Arbiter. She alleged
that she attained a regular status and was entitled to a security of tenure. PCAC questioned the
jurisdiction of the LA, arguing that the case was not covered by the Labor Code but by their church’s
guidelines and by-laws. The LA declared that there was a ER-EE relationship between the parties and
that Benito was illegally dismissed. PCAC elevated the case before the NLRC which reversed the decision
of the LA. NLRC ruled that the non-renewal of Benito’s appointment was considered as ecclesiastical
matter which beyond the ambit of LA’s jurisdiction. However, the CA reversed the NLRC decision and
ruled that the non-renewal of Benito’s appointment was secular in nature and not an ecclesiastical
matter.

               PCAC filed the instant petition, reasserting that the non-renewal or non-extension of Benito’s
tenure is an ecclesiastical matter. They contend that the matter of who are fit to be the congregation’s
ministers, including where and how ministering .is to be conducted, is undoubtedly church or
denomination-related. Thus, a minister or pastor’s fitness to continue in a particular ministry or
congregation is an ecclesiastical affair over which our labor tribunals have no jurisdiction. PCAC also
maintained that Benito continues to be a licensed minister of CAMACOP. It was only her relationship
with PCAC, one of CAMACOP’s local churches, that was severed. Thus, Benito is bound to be reassigned
to other local churches under CAMACOP. PCAC also contend that Benito’s reliance on Pastor Austria
case was misplaced since the cause of termination was not related to the pastor’s ministry while in this
case, Benito’s appointment was due to enforcement of denominational rules. Lastly. PCAC argued that
its ministers having SSS, PhilHealth, and Pag-ibig was only its way to look after them since their ministers
also have families to tend to, and were not exempted from the perils of life.

               On the other hand, Benito echoed the reasoning of the CA’s reasoning that religious
organizations are clearly covered by our Labor Code on termination of employment, and while the case
involves church and its religious minister as regards an internal church policy, it automatically confer the
issue with religious significance

ISSUE:

Is the non-renewal of Benito’s appointment an ecclesiastical matter, which should not be interfered by
the State?

RULING:

Yes

               The Court reiterated that religious associations can be employers for whom religious ministers
often perform dual roles. They not only minister to the spiritual needs of their members in most
instances, but also take on administrative functions in their organizations. The sole concern of the Court
here is whether or not the matter at hand is an ecclesiastic matter over which our labor tribunals are
deprived of jurisdiction.

               The Court pointed out that the Austria case is different from the case at bar. The Adventist
minister in Austria case was removed for alleged misappropriation of denominational funds, willful
breach of trust, serious misconduct and other grounds found in the Labor Code. While the said grounds
for termination may reflect on a minister’s fitness to continue as such, the facts inAustria case indicated
that the grounds for the minister’s dismissal from service were secular in nature. Furthermore, the
Seventh-day Adventist Church in that case admitted before the Labor Arbiter that the minister was its
employee, only to belatedly raise the issue of jurisdiction on appeal. In contrast, PCAC already
questioned the Labor Arbiter’s jurisdiction at the inception of this case.

Petitioners’ contention that there was no dismissal to speak of and the matter concerns their right to
transfer or reassign one of their licensed ministers was well taken by the Court. The Court found that the
claimed right to be infused with religious color because it bears down on the relationship of a church
and its members in faith-based matters. If a church or religious association has the sole prerogative to
exclude members perceived to be unworthy in light of its doctrinal standards, all the more does it have
sole prerogative in determining who are best fit to minister to its members in activities attached with
religious significance.

The Court also disagreed with the CA’s interpretation that the non-renewal of Benito’s appointment was
due to her inefficiency as an administrative officer for her ministry and, thus, purely secular. This
conclusion ignored the significance of Benito’s position under contention, as the Head of Pastoral Care
and Membership, formerly known as Membership and Evangelism Ministry. It also overlooked the fine
line between efficiency and effectiveness. Here, the basis of the non-renewal of Benito’s appointment
was her failure to “share” with new attendees, her inaction on the death of a member, and several other
administrative lapses that impact on the conduct of PCAC’s ecclesiastical activities, such as evangelism,
baptism and Sunday praise or worship activities.

               Guided by the foregoing, the Court held that the termination of a religious minister’s
engagement at a local church due to administrative lapses, when it relates to the perceived effectivity of
a minister as a charismatic leader of a congregation, is a prerogative best left to the church affected by
such choice. If a religious association enacts guidelines that reserve the right to transfer or reassign its
licensed ministers according to what it deems to be for a particular congregation, ministry or
undertaking in pursuit of its mission, then the State cannot validly interfere.
29. San Miguel Corporation vs Etcuban

❖ Reasonable Causal Connection Rule”; Under the “reasonable causal connection rule,” if there is a
reasonable causal connection between the claim asserted and the employer-employee relations, then
the case is within the jurisdiction of the labor courts.

❖ Damages incurred by employees as a result of an alleged fraudulent retrenchment program and the
allegedly defective “contract of termination” are merely the civil aspect of the injury brought about by
their illegal dismissal, and the civil ramifications of their actual claim cannot alter the reality that it is
primordially a labor matter and, as such, is cognizable by labor courts.

❖ If the consent of a contracting party is vitiated by fraud, the contract is not void but, merely, voidable.

Facts: San Miguel Corporation (SMC) informed its Mandaue City Brewery employees that it was suffering
from heavy losses and financial distress which could eventually lead to its total closure. SMC offered its
“Retrenchment to Prevent Loss Program” to its employees. The offering of the retrenchment program
was coupled with an unsolicited advise from SMC that it would be in the best interest of the affected
employees to avail of the said program since, by doing so, they would be able to obtain their
retrenchment benefits and privileges with ease. SMC admonished its employees that their failure to
avail of the retrenchment program might lead to difficulty in following-up and obtaining their separation
pay from SMCʼs main office in Manila. After their inclusion in the retrenchment program, respondents
were given their termination letters and separation pay. In return, respondents executed “receipt and
release” documents in favor of SMC. Sometime in May of 1986, respondents got hold of an SMC
publication allegedly revealing that SMC was never in financial distress during the time when they were
being retrenched but was, in fact, enjoying a growth in sales. Respondents filed a complaint before the
NLRC for the declaration of nullity of the retrenchment program. In their complaint, respondents alleged
that they were former regular employees of SMC who were deceived into severing their employment
due to SMCʼs concocted financial distress story and fraudulent retrenchment program. Those who were
retrenched in 1983, at the very latest, had only until 1987 to institute a complaint against SMC. The
records will show that all the above captioned cases were filed in 1988. Their cases were dismissed, not
because of lack of jurisdiction, but because their cause of action already prescribed, the cases having
been filed after the three-year prescriptive period. the contract of termination which plaintiffs were
allegedly induced to sign is not void from the beginning. At most, such contract is voidable, plaintiffsʼ
consent thereto being allegedly vitiated by fraud and deceit. Defendant corporation inculcated into the
minds of defendants the worry of non-recovery of their benefits in the event defendant corporation
closes down, induced plaintiffs to accept the “offer of retrenchment.” Thereupon, they were paid their
so-called “separation pay.” defendant corporation never suffered any business reverses or losses in its
operation.” When the consent of one of the contracting parties is vitiated by fraud or deceit, the
resulting contract is only voidable or annulable, not void or inexistent. A scrutiny of the allegations of
the present complaint reveals that plaintiffsʼ cause of action is not actually based on an employer-
employee relationship between the plaintiffs and the defendants.

Issue: Whether or not the contract of termination of services entered into by plaintiffs with defendants
is void from the beginning due to inexistent cause of action under Article 1409 of the Civil Code, places
the case within the jurisdiction of the civil courts.

Ruling: Jurisprudence has developed the “reasonable causal connection rule.” Under this rule, if there is
a reasonable causal connection between the claim asserted and the employeremployee relations, then
the case is within the jurisdiction of our labor courts. In the absence of such nexus, it is the regular
courts that have jurisdiction. Jurisprudence has evolved the rule that claims for damages under
paragraph 4 of Article 217, to be cognizable by the Labor Arbiter, must have a reasonable causal
connection with any of the claims provided for in that article. Only if there is such a connection with the
other claims can the claim for the damages be considered as arising from employer-employee relations.
Illegally dismissed employee has only a single cause of action although the act of dismissal may be a
violation not only the Labor Code but also of the Civil Code. The fact that SMC was never in financial
distress does not, in any way, affect the cause of their “contract of termination.” Rather, the fraudulent
representations of SMC only affected the consent of respondents in entering into the said contract. If
the consent of a contracting party is vitiated by fraud, the contract is not void but, merely, voidable.
WHEREFORE, premises considered, the Decision of the Court of Appeals dated 16 May 1996 and its
Resolution dated 14 November 1996 are hereby REVERSED and SET ASIDE and the Resolution dated 21
June 1994 of the Regional Trial Court of Cebu, Branch 19, in CEB15310, REINSTATED.
30. Kawachi v. Del Quero
GR No. 163768
Labor Relations: Jurisdiction

Facts:

On August 10, 2002, petitioners scolded private respondent in front of many people about the way she
treated the customers of the pawnshop and afterwards terminated private respondent’s employment as
a pawnshop clerk. Thereafter, private respondent, in an affidavit-complaint,charged the petitioner with
illegal dismissal, non-execution of a contract of employment, violation of the minimum wage law, and
non-payment of overtime pay. The complaint was filed before the NLRC. The private respondent also
filed an action for damages against the petitioners before the MeTC.

Petitioners moved for the dismissal of the complaint on the grounds of lack of jurisdiction and forum-
shopping or splitting causes of action. At first, the MeTC granted petitioners’ motion and ordered the
dismissal of the complaint for lack of jurisdiction. Upon private respondent’s motion, the MeTC
reconsidered and set aside the order of dismissal. It ruled that no causal connection appeared between
private respondent’s cause of action and the employer-employee relations between the parties.

The RTC held that private respondent’s action for damages was based on the alleged tortuous acts
committed by her employers and did not seek any relief under the Labor Code.

Issue:

Whether the regular courts lack jurisdiction in the case at bar.

Held:

Yes. The regular courts lack jurisdiction in the case at bar.

In the instant case, the allegations in private respondent’s complaint for damages show that her injury
was the offshoot of petitioners’ immediate harsh reaction as her administrative superiors to the
supposedly sloppy manner by which she had discharged her duties. Petitioners’ reaction culminated in
private respondent’s dismissal from work in the very same incident. The incident on 10 August 2002
alleged in the complaint for damages was similarly narrated in private respondent’s Affidavit-Complaint
supporting her action for illegal dismissal before the NLRC. Clearly, the alleged injury is directly related
to the employer-employee relations of the parties.

Where the employer-employee relationship is merely incidental and the cause of action proceeds from a
different source of obligation, the Court has not hesitated to uphold the jurisdiction of the regular
courts. Where the damages claimed for were based on tort, malicious prosecution, or breach of
contract, as when the claimant seeks to recover a debt from a former employee or seeks liquidated
damages in the enforcement of a prior employment contract, the jurisdiction of regular courts was
upheld. The scenario that obtains in this case is obviously different. The allegations in private
respondent’s complaint unmistakably relate to the manner of her alleged illegal dismissal.

31. EDUARDO G. EVIOTA, petitioner, vs. THE HON. COURT OF APPEALS, THE HON. JOSE BAUTISTA,
Presiding Judge of Branch 136, Regional Trial Court of Makati, and STANDARD CHARTERED BANK,
respondents

G.R. No. 152121                |              July 29, 2003

FACTS:

Sometime on January 26, 1998, the respondent Standard Chartered Bank and petitioner Eduardo G.
Eviota executed a contract of employment under which the petitioner was employed by the respondent
bank as Compensation and Benefits Manager, VP (M21). However, the petitioner abruptly resigned from
the respondent bank barely a month after his employment and rejoined his former employer.

On December 22, 1997, Eviota began negotiating with the Bank on his possible employment with the
latter. Taken up during these negotiations were not only his compensation and benefit package, but also
the nature and demands of his prospective position. The Bank made sure that Eviota was fully aware of
all the terms and conditions of his possible job with the Bank.

The bank alleges that his resignation, which did not comply with the 30-day prior notice rule under the
law and under the Employment Contract, was so unexpected that it disrupted plans already in the
pipeline, aborted meetings previously scheduled among Bank officers, and forced the Bank to hire the
services of a third party to perform the job he was hired to do.

Aside from causing no small degree of chaos within the Bank by reason of his sudden resignation, Eviota
made off with a computer diskette and other papers and documents containing confidential information
on employee compensation and other Bank matters.

The bank alleges that in his attempts to justify his hasty departure from the Bank and conceal the real
reason for his move, Eviota has resorted to falsehoods derogatory to the reputation of the Bank. In
particular, he has been maliciously purveying the canard that he had hurriedly left the Bank because” it
had failed to provide him support. His untruthful remarks have fairly depicted the Bank as a contract
violator and an undesirable employer thus damaging the Bank’s reputation and business standing.

Thus the Bank filed a complaint for damages against Eviota with the trial court.
Petitioner filed a motion to dismiss the complaint on the ground that the action for damages of the
respondent bank was within the exclusive jurisdiction of the Labor Arbiter under paragraph 4, Article
217 of the Labor Code of the Philippines, as amended. The petitioner averred that the respondent
bank’s claim for damages arose out of or were in connection with his employer-employee relationship
with the respondent bank or some aspect or incident of such relationship. The respondent bank
opposed the motion, claiming that its action for damages was within the exclusive jurisdiction of the trial
court. Although its claims for damages incidentally involved an employer- employee relationship, the
said claims are actually predicated on the petitioner’s acts and omissions which are separately,
specifically and distinctly governed by the New Civil Code.

ISSUE:

Whether or not the trial court has jurisdiction over the Bank’s complaint for damages

RULING:

Article 217 of the Labor Code of the Philippines, reads:

ART. 217. Jurisdiction of Labor Arbiters and the Commission.— (a) Except as otherwise provided under
this Code the Labor Arbiters shall have original and exclusive jurisdiction to hear and decide within thirty
(30) calendar days after the submission of the case by the parties for decision without extension, even in
the absence of stenographic notes, the following cases involving all workers, whether agricultural or
nonagricultural:

1.       Unfair labor practice cases;

2.       Termination disputes;

3.       If accompanied with a claim for reinstatement, those cases that workers may file involving wages,
rates of pay, hours of work and other terms and conditions of employment;

4.       Claims for actual, moral, exemplary and other forms of damages arising from the employer-
employee relations.

Case law has it that the nature of an action and the subject matter thereof, as well as which court has
jurisdiction over the same, are determined by the material allegations of the complaint and the reliefs
prayed for in relation to the law involved.

Not every controversy or money claim by an employee against the employer or vice-versa is within the
exclusive jurisdiction of the labor arbiter. A money claim by a worker against the employer or vice-versa
is within the exclusive jurisdiction of the labor arbiter only if there is a “reasonable causal connection”
between the claim asserted and employee-employer relation. Absent such a link, the complaint will be
cognizable by the regular courts of justice.

Actions between employees and employer where the employer-employee relationship is merely
incidental and the cause of action precedes from a different source of obligation is within the exclusive
jurisdiction of the regular court.

In this case, the private respondent’s first cause of action for damages is anchored on the petitioner’s
employment of deceit and of making the private respondent believe that he would fulfill his obligation
under the employment contract with assiduousness and earnestness. The petitioner volte face when,
without the requisite thirty-day notice under the contract and the Labor Code of the Philippines, as
amended, he abandoned his office and rejoined his former employer; thus, forcing the private
respondent to hire a replacement. The private respondent was left in a lurch, and its corporate plans
and program in jeopardy and disarray. Moreover, the petitioner took off with the private respondent’s
computer diskette, papers and documents containing confidential information on employee
compensation and other bank matters. On its second cause of action, the petitioner simply walked away
from his employment with the private respondent sans any written notice, to the prejudice of the
private respondent, its banking operations and the conduct of its business. Anent its third cause of
action, the petitioner made false and derogatory statements that the private respondent reneged on its
obligations under their contract of employment; thus, depicting the private respondent as unworthy of
trust.

It is evident that the causes of action of the private respondent against the petitioner do not involve the
provisions of the Labor Code of the Philippines and other labor laws but the New Civil Code. Thus, the
said causes of action are intrinsically civil. There is no causal relationship between the causes of action of
the private respondent’s causes of action against the petitioner and their employer-employee
relationship. The fact that the private respondent was the erstwhile employer of the petitioner under an
existing employment contract before the latter abandoned his employment is merely incidental. In fact,
the petitioner had already been replaced by the private respondent before the action was filed against
the petitioner.
32. INDOPHIL TEXTILE MILLS, INC., Petitioner,vs. ENGR. SALVADOR ADVIENTO, Respondents. G.R. No.
171212 August 4, 2014 DECISION

Facts: Petitioner Indophil Textile Mills, Inc. is a domestic corporation engaged in the business of
manufacturing thread for weaving. On August 21, 1990, petitioner hired respondent Engr. Salvador
Adviento as Civil Engineer to maintain its facilities in Lambakin, Marilao, Bulacan. On August 7, 2002,
respondent consulted a physician due to recurring weakness and dizziness. Few days later, he was
diagnosed with Chronic Poly Sinusitis, and thereafter, with moderate, severe and persistent Allergic
Rhinitis. Accordingly, respondent was advised by his doctor to totally avoid house dust mite and textile
dust as it will transmute into health problems. Distressed, respondent filed a complaint against
petitioner with the National Labor Relations Commission (NLRC), San Fernando, Pampanga, for alleged
illegal dismissal and for the payment of backwages, separation pay, actual damages and attorney’s fees.
Subsequently, respondent filed another Complaint with the Regional Trial Court (RTC) of Aparri,
Cagayan, alleging that he contracted such occupational disease by reason of the gross negligence of
petitioner to provide him with a safe, healthy and workable environment. Respondent alleged that as
part of his job description, he conducts regular maintenance check on petitioner’s facilities including its
dye house area, which is very hot and emits foul chemical odor with no adequate safety measures
introduced by petitioner. According to respondent, the air washer dampers and all roof exhaust vests
are blown into open air, carrying dust thereto. Concerned, respondent recommended to management
to place roof insulation to minimize, if not, eradicate the health hazards attendant in the work place.
However, said recommendation was turned down by management due to high cost. In reply, petitioner
filed a Motion to Dismiss on the ground that: (1) the RTC has no jurisdiction over the subject matter of
the complaint because the same falls under the original and exclusive jurisdiction of the Labor Arbiter
(LA) under Article 217(a)(4) of the Labor Code; and (2) there is another action pending with the Regional
Arbitration Branch III of the NLRC in San Fernando City, Pampanga, involving the same parties for the
same cause. On December 29, 2003, the RTC issued a Resolution denying the aforesaid Motion and
sustaining its jurisdiction over the instant case. It held that petitioner’s alleged failure to provide its
employees with a safe, healthy and workable environment is an act of negligence, a case of quasi-delict.
As such, it is not within the jurisdiction of the LA under Article 217 of the Labor Code. On February 9,
2004, petitioner filed a motion for reconsideration thereto, which was likewise denied in an Order
issued on even date. Petitioner then filed a Petition for Certiorari with the CA on the ground that the
RTC committed grave abuse of discretion amounting to lack or excess of jurisdiction in upholding that it
has jurisdiction over the subject matter of the complaint despite the broad and clear terms of Article 217
of the Labor Code, as amended.

After the submission by the parties of their respective Memoranda, the CA rendered a Decision27 dated
May 30, 2005 dismissing petitioner’s Petition for lack of merit. Petitioner filed a Motion for
Reconsideration which was nevertheless denied for lack of merit in the CA’s Resolution dated January
10, 2006. Whether or not the RTC has jurisdiction over the subject matter of respondent’s complaint
praying for moral damages, exemplary damages, compensatory damages, anchored on petitioner’s
alleged gross negligence in failing to provide a safe and healthy working environment for respondent.
While we have upheld the present trend to refer worker-employer controversies to labor courts in light
of the aforequoted provision, we have also recognized that not all claims involving employees can be
resolved solely by our labor courts, specifically when the law provides otherwise. For this reason, we
have formulated the "reasonable causal connection rule," wherein if there is a reasonable causal
connection between the claim asserted and the employer-employee relations, then the case is within
the jurisdiction of the labor courts; and in the absence thereof, it is the regular courts that have
jurisdiction. Jurisprudence has evolved the rule that claims for damages under Article 217(a)(4) of the
Labor Code, to be cognizable by the LA, must have a reasonable causal connection with any of the claims
provided for in that article. Only if there is such a connection with the other claims can a claim for
damages be considered as arising from employer-employee relations. In the case at bench, we find that
such connection is nil. Respondent’s claim for damages is specifically grounded on petitioner’s gross
negligence to provide a safe, healthy and workable environment for its employees −a case of quasi-
delict. This is easily ascertained from a plain and cursory reading of the Complaint, which enumerates
the acts and/or omissions of petitioner relative to the conditions in the workplace.

Ruling: The cause of action is based on a quasi-delictor tort, which has no reasonable causal connection
with any of the claims provided for in Article 217, jurisdiction over the action is with the regular courts.
Injury and damages were allegedly suffered by respondent, an element of quasi-delict. The previous
contract of employment between petitioner and respondent cannot be used to counter the element of
"no pre-existing contractual relation" since petitioner’s alleged gross negligence in maintaining a
hazardous work environment cannot be considered a mere breach of such contract of employment, but
falls squarely within the elements of quasi-delictunder Article 2176 of the Civil Code since the negligence
is direct, substantive and independent. Where the resolution of the dispute requires expertise, not in
labor management relations nor in wage structures and other terms and conditions of employment, but
rather in the application of the general civil law, such claim falls outside the area of competence of
expertise ordinarily ascribed to the LA and the NLRC. Guided by the aforequoted doctrines, we find no
reason to reverse the findings of the CA. The RTC has jurisdiction over the subject matter of
respondent's complaint praying for moral damages, exemplary damages, compensatory damages,
anchored on petitioner's alleged gross negligence in failing to provide a safe and healthy working
environment for respondent.

You might also like